Download Major Model Test Keys-26-9-2015 1.D 2.A 3.C 4.D 5.D 6.B 7.B 8.B 9

Document related concepts

Sjögren syndrome wikipedia , lookup

List of medical mnemonics wikipedia , lookup

Transcript
Major Model Test Keys-26-9-2015
1.D
2.A
3.C
4.D
5.D
6.B
7.B
8.B
9.B
10.D 11.D 12.B 13.D
14.D 15.D 16.A 17.B 18.C 19.A 20.A 21.B 22.D 23.C 24.D 25.D 26.A
27.C 28.D 29.A 30.A 31.B 32.D 33.D 34.B 35.C 36.D 37.B 38.C 39.C
40.C 41.C 42.D 43.D 44.C 45.B 46.B 47.C 48.C 49.A 50.C 51.B 52.D
53.D 54.B 55.B 56.B 57.A 58.A 59.C 60.C 61.B 62.A 63.A 64.C 65.A
66.C 67.D 68.C 69.D 70.C 71.D 72.B 73.C 74.A 75.D 76.C 77.A 78.C
79.A 80.B 81.C 82.A 83.B 84.C 85.A 86.D 87.C 88.A 89.B 90.B 91.A
92.D 93.A 94.C 95.A 96.B 97.A 98.D 99.B 100.C 101.D 102.A 103.D 104.C
105.B 106.C 107.C 108.B 109.B 110.C
111.ANS.A
EXPLANTION:Anaplasia is lack of differentiation and is a characteristic feature of most malignant tumours. Depending
upon the degree of differentiation, the extent of anaplasia is also variable i.e. poorly differentiated
malignant tumours have high degree of anaplasia. As a result of anaplasia, noticeable morphological and
functional alterations in the neoplastic cells are observed.
i) Loss of polarity. Normally, the nuclei of epithelial cells are oriented along the basement membrane
which is termed as basal polarity. This property is based on cell adhesion molecules, particularly
selectins. Early in malignancy, tumour cells lose their basal polarity so that the nuclei tend to lie away
from the basement membrane
ii) Pleomorphism. The term pleomorphism means variation in size and shape of the tumour cells. The
extent of cellular pleomorphism generally correlates with the degree of anaplasia. Tumour cells are often
bigger than normal but in some tumours they can be of normal size or smaller than normal
iii) N:C ratio. Generally, the nuclei of malignant tumour cells show more conspicuous changes. Nuclei are
enlarged disproportionate to the cell size so that the nucleocytoplasmic ratio is increased from normal 1:5
to 1:1
iv) Anisonucleosis. Just like cellular pleomorphism, the nuclei too, show variation in size and shape in
malignant tumour cells
v) Hyperchromatism. Characteristically, the nuclear chromatin of malignant cell is increased and coarsely
clumped. This is due to increase in the amount of nucleoprotein resulting in dark-staining nuclei, referred
to as hyperchromatism
vi) Nucleolar changes. Malignant cells frequently have a prominent nucleolus or nucleoli in the nucleus
reflecting increased nucleoprotein synthesis This may be demonstrated as Nucleolar Organiser Region
(NOR) by silver (Ag) staining called AgNOR material.
1
vii) Mitotic figures. The parenchymal cells of poorly-differentiated tumours often show large number of
mitoses as compared with benign tumours and well-differentiated malignant tumours. As stated above,
these appear as either normal or abnormal mitotic figures Normal mitotic figures may be seen in some
non-neoplastic proliferating cells (e.g. haematopoietic cells of the bone marrow, intestinal epithelium,
hepatocytes etc), in certain benign tumours and some low grade malignant tumours; in sections they are
seen as a dark band of dividing chromatin at two poles of the nuclear spindle. Abnormal or atypical
mitotic figures are more important in malignant tumours and are identified as tripolar, quadripolar and
multipolar spindles in malignant tumour cells.
viii) Tumour giant cells. Multinucleate tumour giant cells or giant cells containing a single large and
bizarre nucleus, possessing nuclear characters of the adjacent tumour cells, are another important
feature of anaplasia in malignant tumours
Ref:-PathologyRobbins & Cotran 3rd Edition 2009
112
ANS.C
EXPLANTION:The 100 trillion cells of the body are members of a highly organized community in which the total number
of cells is regulated not only by controlling the rate of cell division but also by controlling the rate of cell
death. When cells are no longer needed or become a threat to the organism, they undergo a suicidal
programmed cell death, or apoptosis. This process involves a specific proteolytic cascade that causes the
cell to shrink and condense, to disassemble its cytoskeleton, and to alter its cell surface so that a
neighboring phagocytic cell, such as a macrophage, can attach to the cell membrane and digest the cell.
In contrast to programmed death, cells that die as a result of an acute injury usually swell and burst due
to loss of cell membrane integrity, a process called cell necrosis. Necrotic cells may spill their contents,
causing inflammation and injury to neighboring cells. Apoptosis, however, is an orderly cell death that
results in disassembly and phagocytosis of the cell before any leakage of its contents occurs, and
neighboring cells usually remain healthy.
Apoptosis is initiated by activation of a family of proteases called caspases. These are enzymes that are
synthesized and stored in the cell as inactive procaspases. The mechanisms of activation of caspases
are complex, but once activated, the enzymes cleave and activate other procaspases, triggering a
cascade that rapidly breaks down proteins within the cell. The cell thus dismantles itself, and its remains
are rapidly digested by neighboring phagocytic cells.
A tremendous amount of apoptosis occurs in tissues that are being remodeled during development. Even
in adult humans, billions of cells die each hour in tissues such as the intestine and bone marrow and are
replaced by new cells. Programmed cell death, however, is normally balanced with the formation of new
cells in healthy adults. Otherwise, the body’s tissues would shrink or grow excessively. Recent studies
suggest that abnormalities of apoptosis may play a key role in neurodegenerative diseases such as
Alzheimer’s disease, as well as in cancer and autoimmune disorders. Some drugs that have been used
successfully for chemotherapy appear to induce apoptosis in cancer cells.
Ref:-Guyton and Hall Textbook of Medical Physiology
by John E. Hall
12th edition published by Elsevier
113.ANS.B
EXPLANTION:The ability to destroy the nucleic acids within the cell undergoing apoptosis includes the destruction of
microbial RNA and DNA and inhibits the spread of infectious agents from the dying cells to uninfected
neighboring cells.
Ref:-Immunology
by Thao Doan, Roger Melvold, Susan Viselli, and Carl Waltenbaugh 2nd edition published by Wolters
Kluwer
114.ANS.A
EXPLANTION:-
2
TNF is secreted by macrophages in response to PAMPs and DAMPs and mediates acute inflammatory
responses by activating endothelial cells and leukocytes. Anti-TNF antibodies and recombinant soluble
TNF receptor-IgG fusion proteins have been highly successful in treating RA.
Ref:-Cellular and Molecular Immunology
by Abul K. Abbas, Andrew H. Lichtman, and Shiv Pillai
7th edition published by Elsevier
115.ANS.C
EXPLANTION:Secondary descriptors are used to refer to a tumor’s morphologic and functional characteristics.
Papillomatosis (choice E) describes frond-like structures. Medullary (choice D) signifies a soft cellular
tumor, whereas scirrhous or desmoplastic implies dense fibrous stroma. Colloid carcinomas (choice A)
secrete abundant mucus. Comedocarcinoma (choice B) is an intraductal neoplasm in which necrotic
material can be expressed from the ducts.
Diagnosis: Breast cancer
Ref:-Rubin’s Pathology: Clinicopathologic Foundations of Medicine
by Raphael Rubin and David S. Strayer
6th edition published by Wolters Kluwer
116.ANS.C
EXPLANTION:The renin-angiotensin system (RAS) or the renin-angiotensin-aldosterone system (RAAS) is a hormone
system that regulates blood pressure and water (fluid) balance.
When blood volume is low, juxtaglomerular cells in the kidneys activate their prorenin and secrete renin
directly into circulation. Plasma renin then carries out the conversion of angiotensinogen released by the
liver to angiotensin I. Angiotensin I is subsequently converted to angiotensin II by the enzyme
angiotensin-converting enzyme found in the lungs. Angiotensin II is a potent vaso-active peptide that
causes blood vessels to constrict, resulting in increased blood pressure. Angiotensin II also stimulates the
secretion of the hormone aldosterone from the adrenal cortex. Aldosterone causes the tubules of the
kidneys to increase the reabsorption of sodium and water into the blood. This increases the volume of
fluid in the body, which also increases blood pressure.
The system can be activated when there is a loss of blood volume or a drop in blood pressure (such as in
hemorrhage or dehydration). This loss of pressure is interpreted by baroreceptors in the carotid sinus. In
alternative fashion, a decrease in the filtrate NaCl concentration and/or decreased filtrate flow rate will
stimulate the macula densa to signal the juxtaglomerular cells to release renin.
If the perfusion of the juxtaglomerular apparatus in the kidney's macula densa decreases, then the
juxtaglomerular cells (granular cells, modified pericytes in the glomerular capillary) release the enzyme
renin.
Renin cleaves a zymogen, an inactive peptide, called angiotensinogen, converting it into angiotensin I.
Angiotensin I is then converted to angiotensin II by angiotensin-converting enzyme (ACE), which is
thought to be found mainly in lung capillaries. One study in 1992 found ACE in all blood vessel
endothelial cells.
Angiotensin II is the major bioactive product of the renin-angiotensin system, binding to receptors on
intraglomerular mesangial cells, causing these cells to contract along with the blood vessels surrounding
them and causing the release of aldosterone from the zona glomerulosa in the adrenal cortex.
Angiotensin II acts as an endocrine, autocrine/paracrine, and intracrine hormone.
Ref:-Rubin’s Pathology: Clinicopathologic Foundations of Medicine
by Raphael Rubin and David S. Strayer
6th edition published by Wolters Kluwer
117.ANS.B
EXPLANTION:Immune thrombocytopenic purpura is an example of type II, antibody-mediated, cytotoxic hypersensitivity
reaction. Trimethoprim and sulfamethoxazole bind to platelets, rendering them antigenic. Antibodies are
3
produced against the modified platelets and bind to them, causing activation of complement system via
the classic pathway. Sequential activation of complements results in formation of membrane attack
complexes, which cause lysis of the platelets.
Ref:- Immunology by David Male, Jonathan Brostoff, David B Roth, and Ivan M Roitt 8th edition published
by Elsevier
118.ANS.C
EXPLANTION:Hepar Lobatum
Focal fibrous septation may cause deep linear clefts in the liver capsule such that the liver acquires an
irregular shape; it is also known as “potato liver”
This anomaly is caused by obliteration of large hepatic veins and confluent parenchymal necrosis, as
occurs in adult-onset syphilis, metastatic breast carcinoma, and Hodgkin’s disease.
This process is not true cirrhosis, because the large surviving regions of tissue may be normal
Supernumerary hepatic lobes, a congenital defect, may have a similar appearance
Ref:-Surgical Pathology of the GI Tract, Liver, Biliary Tract and Pancreas
by Robert D. Odze and John R. Goldblum
2nd edition published by Elsevier
119.ANS.A
EXPLANTION:Phenolic glycolipid - 1 (PGL-1) is a Mycobacterium leprae specific cell wall component. It is an
immunodominant antigen and can induce a strong humoral immune response. IgM antibody levels
against PGL-1 were measured in Thai leprosy patients between October 1992-April 1994 by a
commercially available M. leprae particle agglutination test (MLPA). The percentage of seropositivity was
much higher in newly untreated multibacillary (MB) patients (83.9%) than in paucibacillary (PB) patients
(17.8%). Antibody levels in the MB group varied in the range 32-8,192, whereas they varied in the range
32-256 in the PB group. Patients being treated with multidrug therapy (MDT) were 68.3% and 19.4%
seropositive in the MB and PB groups, respectively. Seropositivities in control serum specimens were
11.3% in active pulmonary tuberculosis patients, 2.6% in dermatologic patients and 4.4% in a healthy
population. In conclusion, the anti-PGL-1 assay using MLPA appears to be a sensitive and specific
diagnostic tool for the diagnosis of MB patients. Additionally, it may provide an alternative to the BI
determination in monitoring MB patients under MDT, and also in the surveillance of such patients after
MDT.Ref:-http://www.ncbi.nlm.nih.gov/pubmed/9253875
120.ANS.C
EXPLANTION:Breast (BR) cancer (CA) susceptibility genes (BRCA1 and BRCA2) encode tumor suppressor proteins
involved in checkpoint functions related to progression of the cell cycle into S phase. BRCA1 and BRCA2
proteins also promote DNA repair by binding to RAD51, a molecule that mediates DNA double-strand
repair breaks. The other choices may be abnormal in neoplasia, but they are not primarily affected by
BRCA1.
Diagnosis: Breast cancer
Ref:-Rubin’s Pathology: Clinicopathologic Foundations of Medicine
by Raphael Rubin and David S. Strayer
6th edition published by Wolters Kluwer
121.ANS.A
EXPLANTION:The yellow fever mosquito (Aedes aegypti) is a mosquito that can spread dengue fever, chikungunya,
and yellow fever viruses, and other diseases. The mosquito can be recognized by white markings on its
legs and a marking in
Ref:-
4
122.ANS.D
EXPLANTION:The transmission of a particular strain of dengue from vector to host is determined by how infective the
virus is in the vector, the level of viral replication in tissues of the vector, and how easily the virus can
disseminate from the midgut to the salivary glands . Infectivity, replication, and dissemination are affected
by both the genotype of the infecting virus as well as a variety of mosquito host factors that modulate
dengue virus infection. Some of the host factors that specifically influence dengue infection and
transmission are Aedes vector innate immune system response to viral infection and vector immune
response to co-infection of dengue virus and the parasitic bacterium wolbachia. Competitive strain
competition between different dengue viruses also affects how well individual strains are transmitted
during co-infection and help determine which strains of dengue are maintained in a given host population
Ref:123.ANS.D
EXPLANTION:World Rabies day:September 28
• First successful human antirabies vaccination performed by: Louis Pasteur (1883)
• Serumantibodies take 7 days to appear after vaccination (Maximum levelof Immunityachieved in days)
• Besf prophylaxis ofRabies in exposed persons: Combined Vaccine and Immunoglobulin/ Serum
treatment
• Anti Rabies serum:
Horse Antirabies Serum: 40 IU/ kg on Day 0 (50% in Wound, 50%i.m)
Human Rabies Immunoglobulin: 20 IU/kg (partly in wound, rest i.m gluteal)
Serum Sickness with Horse Serum: 15-45%
• Persons under Antirabic treatment should avoid:
Alcohol (during and 1 month after treatment)
Undue physical and mental strain and late nights
Corticosteroids and other immunosuppressive agents
Intramuscular injections ofCell Culture andPurified Duck Embryo Vaccines: Deltoid (not in Buttocks)
Volume of intradermal dose of Rabies Vaccine is l/5th of intramuscular dose
Sitesfor intradermal rabies vaccines: Deltoid, Lateral thigh, Suprascapular region, Lower quadrant of
abdomen
Booster injections in Pre-exposure prophylaxis: at intervals of 2 years
Immunisation of Dogs: Primary Immunisation at 3-4 months and boosters at regular intervals
BPL inactivated NTV: Singledose5ml for dogs (3ml for cats),revaccination after 6 months, subsequently
every
year
Modified Live Virus Vaccine: Single dose 3ml, boosters every 3 years
• Most logical and cost effective approach forcontrol ofUrban Rabies: Elimination ofstray dogs and swift
mass immunisation
Atleast 80% of entire dog Population of the area must be immunized.
Pre-exposure rabies vaccination consists of three full intramuscular (i.m.) doses of cell-culture- or
embryonated-egg-based vaccine given on days 0, 7 and 21 or 28 (a few days’ variation in the timing is
not important). For adults, the vaccine should always be administered in the deltoid area of the arm; for
young children (under 1 year of age), the anterolateral area of the thigh is recommended. Rabies vaccine
should never be administered in the gluteal area: administration in this manner will result in lower
neutralizing antibody titres.
To reduce the cost of cell-derived vaccines for pre-exposure rabies vaccination, intradermal (i.d.)
vaccination in 0.1-ml volumes on days 0, 7 and either 21 or 28 may be considered. This method of
administration is an acceptable alternative to the standard intramuscular administration, but it is
technically more demanding and requires appropriate staff training and qualified medical supervision.
Concurrent use of chloroquine can reduce the antibody response to intradermal application of cell-culture
rabies vaccines. People who are currently receiving malaria prophylaxis or who are unable to complete
5
the entire three-dose pre-exposure series before starting malarial prophylaxis should therefore receive
pre-exposure vaccination by the intramuscular route.
Ref:124.ANS.B
EXPLANTION:Rabies occur due to: Animal bites (dogs, cats, monkeys, cow, goat, sheep, buffalo, horses, bats, foxes,
jackals, hyenas
EXCEPT RAT BITE and HUMAN BITE)
• Human bites are likely to transmit: Streptococcusviridans, Staphylococcusaureus, Eikenellacorrodens,
Hemophilus
influenzae, Fusobacterium nucleatum, Prevotella, Porphyromonas, Peptostreptococcus, HIV, Hepatitis B
• Rat bite can transmit: Streptobacillus monliformis, Spirillum minor
Ref:125.ANS.C
EXPLANTION:Bite wounds should notbeimmediately sutured to prevent additional trauma, which may help spread of the
rabies virus deeper into the tissues.
Ifsuturingis necessary, it should bedone 24-48 liours later, with minimal possible stitches, under cover of
anti-rabies serum locally.
Rabies vaccinewasfirst developed by:Louis Pasteur (and Emile Roux)
Strain of Human Diploid Cell Vaccine: Attenuated Pitman-Moore L503 strain
Induced Coma Treatment: In 2005, the case of Jeanna Giese, a girl of 15 who survived acute,
unvaccinated rabies was reported, indicating the successful treatment of rabies through induction of a
coma
Ref:126.ANS.B
EXPLANTION:Period of communicability:
Cliickenpox: 1-2 days before to 4 - 5 days after appearance of rash
Measles: 4 days before to 5 days after appearance of rash
Diphtheria: 14 - 28 days from disease onset
Poliomyelitis: 7-10 days before and after onset of symptoms
CHICKENPOX:
• Also known as 'Varicella'
• Causative agent: Varicella zoster virus [Human (alpha) Herpes Virus - 3]
Secondary Attack rate: 90%
• Incubation period: 14-16 days
Rash:
Chickenpox rash Dew drop on rose petal appearance
Centripetal distribution Pleomorphic rash
Smallpox rash
Centrifugal distribution
Non-pleomorphic
• MClate complication of Chickenpox: Shingles (caused by reactivation of the virus decades after the
initial episode of
chickenpox)
• Aspirin must notbegiven tochildren with cliickenpox: Risk of Reye's Syndrome
• Strain of Live attenuated Chickenpox Vaccine: OKA strain
• Congenital Varicella: Most threatening if transmitted in 1sttrimester of pregnancy
Ref:-[Ref. Park21/epl34, Park22/e pl36J
6
127.ANS.D
EXPLANTION:Epidemiological Reasons/Basis For Smallpox Eradication:
• No known animal reservoir
• No long term carrier state
• Infection provides lifelong immunity
• Case detection simple due to characteristic rash
• Subclinical cases did not transmit the disease
• A highly effective vaccine was available
• International cooperation
Ref:-IRef. Park 21/e pl32, Park 22/e pl35]
128.ANS.C
EXPLANTION:Period ofCommunicability: 4 days beforeand 5 days after the appearance of rash (Rash: Retro-auricular
origin)
Measles is highly infectious during pro-dromal period and during eruption
Period of communicability declines rapidly after appearance of rash
• Measles has no second attacks (life long immunity seen)
Ref:-(Ref Park 21/e pl36-40, Park 22/e pl37-39]
129.ANS.B
EXPLANTION:PATHOGENESIS AND IMMUNITY
The initial event in influenza is infection of the respiratory epithelium with influenza virus acquired from
respiratory secretions of acutely infected individuals. In all likelihood, the virus is transmitted via aerosols
generated by coughs and sneezes, although hand-to-hand contact, other personal contact, and even
fomite transmission may take place. Experimental evidence suggests that infection by a small-particle
aerosol (particle diameter, <10 μm) is more efficient than that by larger droplets. Initially, viral infection
involves the ciliated columnar epithelial cells, but it may also involve other respiratory tract cells, including
alveolar cells, mucous gland cells, and macrophages. In infected cells, virus replicates within 4–6 h, after
which infectious virus is released to infect adjacent or nearby cells. In this way, infection spreads from a
few foci to a large number of respiratory cells over several hours. In
experimentally induced infection, the incubation period of illness has ranged from 18 to 72 h, depending
on the size of the viral inoculum. Histopathologic study reveals degenerative changes, including
granulation, vacuolization, swelling, and pyknotic nuclei, in infected ciliated cells. The cells eventually
become necrotic and desquamate; in some areas, previously columnar epithelium is replaced by
flattened and metaplastic epithelial cells. The severity of illness is correlated with the quantity of virus
shed in secretions; thus the degree of viral replication itself may be an important factor in pathogenesis.
Despite the frequent development of systemic signs and symptoms such as fever, headache, and
myalgias, influenza virus has only rarely been detected in extrapulmonary sites (including
the bloodstream). Evidence suggests that the pathogenesis of systemic symptoms in influenza may be
related to the induction of certain cytokines, particularly tumor necrosis factor α, interferon α, interleukin 6,
and interleukin
8, in respiratory secretions and in the bloodstream.
Ref:-Harrison 18th
130.ANS.A
EXPLANTION:Congenital Rubella Maternal infection in early pregnancy can lead to fetal infection, with resultant
congenital rubella. The classic signs of congenital rubella are cataract, heart disease,deafness, and
7
myriad other defects The most important factor in the pathogenicity of rubella virus for the fetus is
gestational age at the time of infection. Maternal infection during the first trimester
leads to fetal infection in ∼50% of cases; maternal infection early in the second trimester leads to fetal
infection in about one-third of cases. Fetal malformations not only are more common after maternal
infection in the first trimester but also tend to be more
severe and to involve more organ systems.Live attenuated rubella vaccine was licensed in 1969,
7 years after rubella virus was first isolated in culture.This vaccine was developed as a strategy to prevent
congenital rubella by ensuring that very few pregnant women would be susceptible and that there would
be little circulating wild-type virus. Rubella vaccine induces seroconversion in >95% of recipients. Since
its licensure, there have been no major epidemics in the United States, and the number of cases has
declined by 98%. The vaccine currently licensed in the United States, RA 27/3, is propagated in human
diploid cells and is more immunogenic (particularly with regard to the stimulation of secretory immunity)
than previously licensed vaccines. The present vaccination strategy developed in part when measles was
not being adequately 955 controlled, is to immunize all infants at 12–15 months of age with measlesmumps-rubella (MMR) vaccine and to administer a second dose in early childhood. Rubella vaccine may
also be administered to anyone who is thought to be susceptible to the infection and is not pregnant; it is
particularly important that hospital workers of either sex be immune to rubella so that nosocomial
transmission is avoided. Although there has been little change in the prevalence of immunity to
rubella among women of childbearing age (∼80%), the incidence of congenital rubella is extremely low,
with fewer than 10 cases annually. It is likely that, although antibody may be undetectable years after
immunization, protection against infection—possibly due to cell-mediated immunity—is the rule. At
present, there is little if any evidence of significant waning of clinically important immunity to rubella with
time
Ref:-Harrison 18th
131.ANS.A
EXPLANTION:CENTRALLY DISTRIBUTED
MACULOPAPULAR ERUPTIONS
Centrally distributed rashes, in which lesions are primarily truncal, are the most common type of eruption.
The rash of rubeola (measles) starts at the hairline 2–3 days into the illness and moves down the body,
sparing the palms and soles It begins as discrete erythematous lesions, which become confluent as the
rash spreads. Koplik’s spots (1- to 2-mm white or bluish lesions
with an erythematous halo on the buccal mucosa) are pathognomonic for measles and are generally
seen during the first 2 days of symptoms. They should not be
confused with Fordyce’s spots (ectopic sebaceous glands), which have no erythematous halos and are
found in the mouth of healthy individuals. Koplik’s spots may briefly overlap with the measles exanthem.
Ref:-Harrison 18th
132.ANS.B
EXPLANTION:DEMOGRApHY
Definition
Scientific study of human population
Population size
Composition
Distribution
Demographic Processes
1 Fertility
2. Mortality
3. Marriage
4. Migration
5. Socral mobility
Types of demography
8
Formal demography Measurement of population process
Social demography Also analyse the relationship between economic, social, cultural and
biological processess influencrng a population
. • Biodemography Science dealing with the integration of biology(BR,DR) and demography
Basic demographic equation
P t+1 =Pt + Birth-Deaths +immigration-Emmigration
Ref:-http://en.wikipedia.org/wiki/Demography#Basic_equation
133.ANS.C
EXPLANTION:Mala N - Estrogen=Ethinyl estradiol
=0.03 mg, Progestrogen=Norgestrel
=0.30mg, No. of tablets
to be used = 4+4,
Combined oral contraceptive pill
(Yuzpe Regimen)
Combined estrogen-progestin contraceptive pills (COC) used in dosage so as to deliver 100 µg of ethinyl
estradiol plus 0.5 mg of levonorgestrel (or 1.0 mg norgestrel) as soon as possible but optimally within 72
hours after unprotected intercourse, twice at 12 hours interval.
As different COC formulations are available, the number of tablets to be used can be decided
Advantages :
• universally available
• women familiar with pills thus find COC more acceptable
Disadvantages :
• confusion about doses in different formulations
• contraindications to estrogen use in some women
• higher probability of side-effects as compared to LNG
Ref:-http://www.aiims.edu/aiims/events/Gynaewebsite/ec_site/manual/1_2_1.htm
134.ANS.C
EXPLANTION:A copper IUD (ATC G02BA) (also intrauterine device, IUD, copper-T, or coil) is a type of intrauterine
device. The copper IUD is a type of long-acting reversible contraception and is one of the most effective
forms of birth control
The failure rates for different models vary between 0.1 and 2.2% after 1 year of use. The T-shaped
models with a surface area of 380mm² of copper have the lowest failure rates. The TCu 380A (ParaGard)
has a one year failure rate of 0.8% and a cumulative 12-year failure rate of 2.2%. Over 12 years of use,
the models with less surface area of copper have higher failure rates. The TCu 220A has a 12 year failure
rate of 5.8%, whereas the TCu 380A has a 12 year failure rate of 2.2%
Ref:-http://en.wikipedia.org/wiki/IUD_with_copper
135.ANS.A
EXPLANTION:Emergency contraception'
1. Yuzpe and Lancee method
2. Mini pitls(POP)
3. High dose estrogen
4. Antiprogestogen(Mifepristone RU 486)
·5. IUD insertion
Yuzpe and. Lancee method
Current recommendation(pills with 30mcg estrogen)~ 4 pills immediately followed by 4 pills
9
12 hours later
Standard method (pills with 50mcg estrogen)~ 2 pills immediately followed by 2 pillS 12
hours later
Pills with 200mcg estrogen~
1 pill immediately followed
by 1 pill 12 hours later
Regimen completed within 72 hours of coitus
Pregnancy test should be carried out if period is >3 days late
Mini pills(POPj
Levonorgestrel 0.75mg tablets
1 pill immediately(within 72 hrs) followed by 1 pill 12 hours later
High dose estrogen
Esrogen 5mg 00 X 5 days
Mifepristone RU-486
10 mg stat within 72 hours
IUD insertion
Mus,t be inserted within 5 days
Non contraceptive benefits of IUD
Synechiolysis in Ashermans syndrom
Reduces endometrial cancer
Treatment of menorrhagia (LNG-IUD)
HRT (LNG-IUD)
Adjuvant therapy for Tamoxifen (LNG-IUD)
Non contraceptive benefits of OOP
PCOD
Endometriosis
• Adenomyosis
Anaemia
Dysmenorrhes
Acne
Irregular cycles
DUB
Non contraceptive benefits of Barrier methods
Prevention of HIV
Prevention of STls
Ref:-Mahajan and Gupta Textbook of Preventive and Social Medicine, 4th Edition
136.ANS.E
EXPLANTION:A decrease or cessation in sensations of fetal activity may be an indication of fetal distress or death,
though it is not entirely uncommon for a healthy fetus to exhibit such changes, particularly near the end of
a pregnancy when there is considerably less space in the uterus than earlier in pregnancy for the fetus to
move about. Still, medical examination, including a nonstress test, is recommended in the event of any
type of any change in the strength or frequency of fetal movement, especially a complete cease; most
midwives and obstetricians recommend the use of a kick chart to assist in detecting any changes. Fetal
distress or death can be confirmed or ruled out via fetoscopy/doptone, ultrasound, and/or electronic fetal
monitoring. If the fetus is alive but inactive, extra attention will be given to the placenta and umbilical cord
during ultrasound examination to ensure that there is no compromise of oxygen and nutrient delivery.
Ref:-http://en.wikipedia.org/wiki/Stillbirth#Prenatal_diagnosis
137.ANS.E
EXPLANTION:-
10
Reducing the maternal mortality by three quarters between 1990 and 2015 is the first target of Goal 5 —
Improving Maternal Health — of the eight Millennium Development Goals; its progress is monitored at
mdgmonitor.org
Ref:-http://en.wikipedia.org/wiki/Maternal_death
138.ANS.A
EXPLANTION:IMR
= No. of deaths < 1 year of age in a year / Total no.of live births in the same year x1000
In India - 58/1000 live births (2004)
Most important and sensitive indicator of level of living
Majority are neonatal deaths (70%)
Largest single age category of mortality
IMR - Different states
Orissa 77
M.P- 79
UP- 72
Rajasthan 67
Kerala 12
Tamilnadu 41
Kamataka 49
IMR-Country figures
Srilanka 12/1000
China 36/1000
Thailand 18/1000
India 58/1000
Ref:-Mahajan and Gupta Textbook of Preventive and Social Medicine, 4th Edition
139.ANS.A
EXPLANTION:The book quoted above gives a comprehensible table for what to do when an unimmunised child of 1 - 5
years comes and when an unimmunized child above 5 years comes
Visit
1 - 5 years
Above 5 years
First Visit
BCG, DPT, OPV
DT, OPV, Mantoux
After 4 weeks
DPT, OPV
DT, OPV, BCG if Mantoux Positive
After 4 weeks
DPT, OPV, Measles
DT, OPV
After 1 year
DPT,OPV
After 2-3 years
DT
In one single visit, a 9 month-old, unimmunized child can be given the following vaccination:
1.Only BCG.
2.BCG, DPT-1, OPV-1.
3.DPT-1, OPV-1, Measles.
4.BCG,DPT-1, OpV-1, Measles.
The answer in this case will be BCG,DPT-1, OpV-1, Measles because of the word single.
11
Tips
Note that only a single dose of Hib is needed (if at all indicated)
Ref:-Immunisation in Practise - Mittal and Kukreja - CBS Publishers Page 19
140.ANS.C
EXPLANTION:Essential Components of RCH Programme
1. Prevention and management of unwanted pregnancy.
2. Maternal care that includes antenatal, delivery and postpartum services.
3. Child survival services for newborns and infants.
4. Management of Reproductive Tract Infection (TRIs) and Sexually Transmitted Infections (STIs).
The Government have power to restrict any unit, and to take samples of effluents and to get them
analysed in Central or State laboratories. Whoever fails to comply with any provision of this Act is
punishable with the imprisonment or with fine or with both. Second or third time breaking of the law is
further punishable. Under the provision of this Act Central Pollution Control Board was established to
fulfill its object.
Major Elements of RCH Programme
A. Reproductive Health Elements
Responsible and healthy sexual behaviour
Interventions to Promote Safe Motherhood
Essential Obstetric Care for All
Prevention of Unwanted Pregnancies: Increase Access to Contraceptives
Emergency Contraceptives
Safe Abortion
Pregnancy and Delivery Services
First Referral Units (FRUs) for Emergency Obstetric Care
Management of RTIs/STDs
Infertility & Gynecological Disorders
Referral facilities by Government /Private Sector for Pregnant Woman at Risk
Reproductive Health Services for Adolescent Health
Global Reproductive Health Strategy
B. Child Survival Element
Essential New Born Care
Prevention and Management of Vaccine Preventable Disease
Urban Measles Campaign
Elimination of Neonatal Tetanus
Cold Chain System
Polio Eradication: Pulse Polio Programmes
Hepatitis B Vaccine
MMR Vaccine
Global Alliance for Vaccine and Immunisation (GAVI)
Diarrhea Control Programme and ORS Programme
Prevention and Control of Vitamin A deficiency among children
Strategy
1. Bottom-up Planning
2. Decentralised Participatory Planning & Implementation
3. Strengthening Infrastructure
4. Integrated Training Package
5. Improved Management
12
Ref:http://www.nihfw.org/NDC/DocumentationServices/NationalHealthProgramme/REPRODUCTIVEANDCHI
LDHEALTH.html
141.ANS.D
EXPLANTION:A positive ultrasound will definitely confirm that a woman is pregnant since the fetus in utero is directly
visualized.
Ref:-Mahajan and Gupta Textbook of Preventive and Social Medicine, 4th Edition
142.ANS.C
EXPLANTION:Leg cramps is caused by the contraction of the gastrocnimeus (leg muscle). Thus, the intervention is to
stretch the muscle by dosiflexing the foot of the affected leg towards the knee.
Ref:-Mahajan and Gupta Textbook of Preventive and Social Medicine, 4th Edition
143.ANS.C
EXPLANTION:The LBW group thus includes both preterm and term small-for-date newborns. About 30% newborns in
India are in the LBW category. The high incidence of LBW in our country is accounted by a high number
of term SFD infants (i.e. children having intrauterine growth retardation). When it comes to mortality
statistics, prematurity and LBW together constitute the first most
common cause of infant death in urban India whereas
it ranks second in the rural population. Goals for 10th
plan is to reduce LBW to 10%.
Ref:-Mahajan and Gupta Textbook of Preventive and Social Medicine, 4th Edition
144.ANS.E
EXPLANTION:Diabetes mellitus often produces Large for date babies or preterm.LBW is less common
Etiology of LBW The causes of low birth weight are not very wellunderstood. In one-third to half, the
cause may remain unknown. In any case, the causes of LBW are related to the health of the mother:
• Factors affecting the general health of the mother—
These include malnutrition, anemia, tuberculosis and
other infective and systemic diseases.
• Factors affecting antenatal health of the mother—
These include pre-eclamptic toxemia, urinary
infection; hypertension, multiple pregnancy, etc.
Heavy infection of the placenta with malarial parasite
can cause LBW in malarious areas
Ref:-Mahajan and Gupta Textbook of Preventive and Social Medicine, 4th Edition
145.ANS.E.EXPLANTION:Components of Antenatal Checkups
Antenatal Measurements/Tests
• Weight measured
• Height measured
• Blood pressure checked
• Blood tested – Hb, blood group, Rh factor, VDRL test
• Urine tested – albumin, sugar
• Abdomen examined
• Breast examined
• Sonogram/Ultrasound done
13
• Delivery date told
• Delivery advice given
• Nutrition advice given.
Ref:-Mahajan and Gupta Textbook of Preventive and Social Medicine, 4th Edition
146.ANS.A
EXPLANTION:Eugenics
Eugenics refers to improvement of the genetic stock of the race. Eugenic aspects of services to the
mother include prevention of births of children with serious genetic disease, e.g. Down’s syndrome,
muscular dystrophies, hemophilia, etc. This may be achieved through nonterminal or terminal
methods of contraception or through medical termination of pregnancy.
Ref:-Mahajan and Gupta Textbook of Preventive and Social Medicine, 4th Edition
147.ANS.A
EXPLANTION:ANTENATAL CARE
An antenatal care should have contact with the health facility as early as possible. This could be either at
home or in the clinic once a month in the first 7 months, twice a month during the eighth month and
weekly in the 9th month. Under the antenatal program, each registered antenatal case is expected to
receive a minimum of 4 physical examinations, of which at least one is to take place at more than 36
weeks gestation. Each antenatal case is also expected to receive at least one home visit prior to delivery.
Ref:-Mahajan and Gupta Textbook of Preventive and Social Medicine, 4th Edition
148.ANS.D
EXPLANTION:Who can be vaccinated with the nasal spray flu vaccine
The nasal spray is approved for use in healthy* people 2 through 49 years of age who are not pregnant.
Who should not be vaccinated with the nasal spray flu vaccine?
Children younger than 2 years
Adults 50 years and older
People with a history of severe allergic reaction to any component of the vaccine or to a previous dose of
any influenza vaccine
People with asthma
Children or adolescents on long-term aspirin treatment.
Children and adults who have chronic pulmonary, cardiovascular (except isolated hypertension), renal,
hepatic, neurologic/neuromuscular, hematologic, or metabolic disorders
Children and adults who have immunosuppression (including immunosuppression caused by medications
or by HIV)
Pregnant women
Moderate or severe acute illness with or without fever is a general precaution for vaccination. GBS within
6 weeks following a previous dose of influenza vaccine is considered a precaution for use of influenza
vaccines.
Ref:-http://www.cdc.gov/flu/about/qa/nasalspray.htm
149.ANS.C
EXPLANTION:Civil Registration System(CRS)
Birth and Death registration system
Birth, Death and marriage is compulsory at their places of occurrence with local registrar
Birth and Death must be registered within 21 days
Marriages must be registered within 30 days
14
Delayed registration for birth/death
After 21 days till 30 days: Late fee
After 30 days till 1 year: Late fee + written permission from district registrar(affidavit)
After 1 year: Late fee + Order of executive magistrate
Ref:-Mahajan and Gupta Textbook of Preventive and Social Medicine, 4th Edition
150.ANS.A
EXPLANTION:Accredited social health activists (ASHAs) are community health workers instituted by the government of
India's Ministry of Health and Family Welfare (MoHFW) as part of the National Rural Health Mission
(NRHM). The mission began in 2005; full implementation was targeted for 2012. Once fully implemented,
there is to be "an ASHA in every village" in India, a target that translates into 250,000 ASHAs in 10 states.
The grand total number of Ashas in India was reported in January 2013 to be 863,506
ASHAs are local women trained to act as health educators and promoters in their communities. The
Indian MoHFW describes them as .health activist(s) in the community who will create awareness on
health and its social determinants and mobilize the community towards local health planning and
increased utilization and accountability of the existing health services.
Their tasks include motivating women to give birth in hospitals, bringing children to immunization clinics,
encouraging family planning (e.g., surgical sterilization), treating basic illness and injury with first aid,
keeping demographic records, and improving village sanitation ASHAs are also meant to serve as a key
communication mechanism between the healthcare system and rural populations.
Ref:-Mahajan and Gupta Textbook of Preventive and Social Medicine, 4th Edition
151.ANS.D
EXPLANTION:UV RADIATION
Occupational health hazard
Arc welding
Reversible
Keratitis (Welder's flash)
Vibration - "White fingers'( 10-500 hz)
Maximum permissible level of Occupational exposure to
radiation 5 rem/ year to whole body.
The effects of UVR on skin depend primarily on wavelength, length of exposure, intensity of exposure,
repetition of exposure, age at time of exposure, site of exposure, and genetic factors of the individual who
is exposed. To have a biologic effect, UVR must be absorbed. Absorbing molecules in the skin are known
aschromophores, and different wavelengths of UVR are absorbed by different chromophores. Among
these are nucleic acids (especially pyrimidine bases), various amino acids in cutaneous proteins, and
lipoproteins in cell membranes. These chromophores are at varying depths in the skin, accounting in part
for the differing photobiologic responses to UVR. To date, there is only imprecise—but improving—
knowledge of how a given photochemical reaction results in a specific biochemical product, thereby
leading to an observable clinical change.
Ultraviolet C Radiation
UVC has no appreciable impact on human health, as it is effectively screened from Earth’s surface by
stratospheric ozone and oxygen. Even with continued ozone depletion, levels of UVC are not expected to
rise. As mentioned, manmade sources of UVC such as germicidal lamps and arc-welding devices have
been only rarely associated with cutaneous pathology as might be predicted, because UVC is effectively
absorbed by the stratum corneum, which is the outermost cutaneous layer.
Ultraviolet B Radiation
UVB acutely induces a cutaneous inflammatory response that is at least partially definable clinically,
histologically, and biochemically. Clinically, erythema or sunburn is the hallmark of acute overexposure to
UVB. The action spectrum for erythema peaks with UVB. Generally, UVB is considered to be 1000-fold
more effective than UVA for induction of erythema. In a human model, 300-nm UVB is 1280-fold more
15
effective at inducing erythema than 360-nm UVA. Erythemogenic doses are usually defined as multiples
of the minimal erythema dose (MED), which is the lowest dose that elicits perceptible erythema. In a
typical fair-skinned individual, the MED might range from 15 to 70 mJ/cm2 for UVB and from 20 to 80
J/cm2 for UVA.
For example, 1 MED of UVB for a typical fair-skinned individual would require 20 minutes of midsummer
exposure in San Diego, whereas 1 MED of UVA would require 2 to 3 hours of exposure. In a day’s time, a
person can receive 15 MEDs of UVB but only 2 to 4 MEDs of UVA. Consequently, although people are
exposed to 10-fold to 100-fold more UVA than UVB, more than 90% of sunlight-induced erythema is
attributable to UVB. The erythema action spectrum, which peaks in UVB, is remarkably similar to the
absorption spectrum of DNA, which suggests that DNA is a principal target chromophore for UVB-induced
erythema and pyrimidine dimer formation. Supporting this is the finding that pyrimidine dimer yields
correlate with erythema.
Sunburn reflects a local vascular reaction. The causes are multifactorial: DNA damage, prostaglandin
activation, cytotoxicity, and other mechanisms are implicated. UVB erythema has its onset 2 to 6 hours
after exposure; it peaks at around 12 to 36 hours and fades after 72 to 120 hours. Acute histologic
changes that accompany UVB exposure include edema with vasodilation of the upper dermal
vasculature137 and endothelial cell swelling, most likely as a result of vasoactive mediators being
released.112 Delayed histologic changes include the appearance of sunburn cells in as little as 30
minutes after exposure. These dyskeratotic cells have enlarged nuclei and vacuolated cytoplasm. Initially,
sunburn cells are localized in the lower one-half of the epidermis; however, after 24 hours, they are also
found in the upper half. These sunburn cells may represent actively cycling and proliferating basal cells
that cannot adequately repair UVR-induced DNA lysosomal damage. Beginning 1 hour after exposure,
stainable Langerhans’ cells (i.e., cutaneous antigen-presenting cells) are reduced by 25%; by 72 hours
after exposure, only 10% remain. In mice that are exposed to repetitive suberythemogenic doses of UVB,
normal numbers of Langerhans’ cells return by 8 days after exposure. Vacuolization of melanocytes is
seen after 1 hour and returns to normal 4 to 24 hours after exposure. Mast cells decrease in number and
granularity within 1 hour, returning to normal after 12 to 72 hours. By 24 to 48 hours after exposure, there
is increased melanin synthesis, epidermal proliferation, and thickening of the stratum corneum.
Functionally, the epidermal permeability barrier diminishes after UVB exposure because of the altered
kinetics of lamellar-body–containing cells in the exposed epidermis.
Biochemical changes that accompany sunburn include increased levels of histamine, which return to
normal within 74 hours. However, histamine is unlikely to be the sole or even the principal mediator of
vasodilation and erythema, because antihistamines are ineffective in preventing sunburn. UVR induces
increased phospholipase activity, with accompanying increases in prostaglandins (PGs). PGD2, PGE2,
PGF, and 12-hydroxyeicosatetraenoic acid are increased in suctioned blister aspirates immediately after
UVB exposure, and peak after 18 to 24 hours. Topical and intradermal indomethacin, which is a
prostaglandin inhibitor, blocks UVB-induced erythema for 24 hours after exposure, thus supporting the
thesis that eicosanoids (PGs and leukotrienes) are significant mediators of UVR-induced inflammation.
UVB stimulates induction of proinflammatory and mutagenic cytokines: tumor necrosis factor-αand
interleukins 6 and 12. UVR generates free radicals in the skin that probably contribute to the sunburn
reaction by causing peroxidative chromosomal, membrane, and protein damage. Topical antioxidants
may mitigate sunburn when they are applied before but not after exposure. Melatonin and vitamin C57
act protectively by scavenging UVR-generated free radicals; however, neither absorbs UVR.
Ultraviolet A Radiation
UVA penetrates more deeply into the skin than does UVB. Whereas 95% of incident UVB is reflected or
absorbed by the epidermis, nearly 50% of UVA reaches the dermis.75 UVA contributes modestly to
sunburn and may cause clinical erythema. Prolonged daily UVA exposure can approach 125 J/cm2,
which significantly exceeds the threshold erythema dose of 20 to 80 J/cm2. Clinically, UVA erythema has
an onset within 4 to 6 hours; it peaks after 8 to 12 hours and fades after 24 to 48 hours. Erythema that
results from UVA may have a distinct pathophysiologic mechanism. UVA-induced erythema may be
caused by keratinocyte cytotoxicity. Histologically, UVA erythema displays more epidermal spongiosis,
fewer sunburn cells, and more dermal changes than does UVB-induced erythema, with a denser and
deeper mononuclear cell infiltrate and more vascular damage.
Ref:-Wilderness Medicine
by Paul S. Auerbach
16
6th edition published by Elsevier
152.ANS.D
EXPLANTION:Coal workers Pneumoconiosis Simple Pneumoconiosis- about 12 yrs of work exposure
Progressive massive fibrosis-Mortality rate among Coal miners- twice the general population
Notifiable disease under Indian mines act(1952) Compensation-Work men compensation act(1959)
Chronic Obstructive Pulmonary Disease (COPD) This term describes three entities that show
considerable clinical overlap:
1.Chronic bronchitis
2.Emphysema
3.Small airways disease
COPD is common and is the fourth leading cause of death worldwide.
Aetiology The main factors are:
1.SMOKING – by far the major risk factor and dose related.
2.α1-antitrypsin deficiency. Reduced levels of this protease inhibitor correlate with lung damage,
especially in smokers.
3.OCCUPATION, especially exposure to dusts, e.g. coal mining.
Patients with respiratory exposure to silica can develop lymphadenopathy, often in association with lung
disease, although occasionally lymphadenopathy is present in isolation. Microscopic examination shows
sclerotic nodules associated with a histiocytic and fibroblastic reaction and birefringent crystals on
examination with polarized light. These lymph nodes often also show anthracosis
Anthracosis is a common finding in mediastinal lymph nodes. Involved nodes typically show black
discoloration on gross examination and numerous histiocytes with carbon particles on microscopic
examination. Lymph nodes draining skin with tattoos may contain histiocytes with pigment that is usually
black. Infrequently, this phenomenon produces clinically significant lymphadenopathy
Ref:-Hematopathology: A Volume in the Series Foundations in Diagnostic Pathology
by Eric D. Hsi and John R. Goldblum
2nd edition published by Elsevier
153.ANS.C
EXPLANTION:Asbestos is the generic name for a group of fibrous silicates widely used in heavy industry for insulation
(particularly in shipbuilding).
There are two distinct forms of asbestos – serpentine (curly and flexible) and amphibole (straight and
stiff). Amphibole fibres (crocidolite and amosite) are more pathogenic than serpentine fibres (principally
chrysotile).
Asbestosis results from chronic exposure.
Other important complications are:
1.Lung cancer
Asbestos workers have an increased risk of lung cancer, especially if they also smoke. Risk of cancer =
risk due to smoking × risk due to asbestos.
2.Pleural pathology
(a)Pleural plaques – these are the most common manifestation of asbestos exposure.
(b)Mesothelioma (Malignant tumour of the pleura.)
This can occur up to 40 years after a brief exposure to asbestos (particularly crocidolite).
Note: There are usually significantly fewer asbestos fibres/bodies in mesothelioma than in asbestosis.
Note: Mesothelioma may also affect the pericardium and peritoneum.
ASBESTOSIS - CHRONIC EFFECTS
CA bronchus
Pleura / peritoneum (Mesothelioma)
CAGIT
Asbestos of 2 types
Serpentine (Chrysollite / Amphibple)
17
Serpentine variety (90%) - hydrated Magnesium silicate
Amphibole type - little magnesium
Amphebole
Crocidolite, Amosite, Amphrophyllite
Disease progresses even after worker removed fron contact
Ref:-Pathology Illustrated
by Robin Reid, Fiona Roberts, and Elaine MacDuff
7th edition published by Elsevier
154.ANS.D
EXPLANTION:Crocidolite - Blue
Amosite - brown
Anthrophyllite -White
Malignant mesothelioma
This primary neoplasm of the pleura is rare, but is an important cause of death in people previously
exposed to asbestos, such as those from the ship building industry. There is a latent interval between
asbestos exposure and disease presentation of between 25 and 45 years. Fibres of crocidolite (blue
asbestos) are thought to be most important in the development of mesothelioma. These asbestos fibres
become trapped in the lung following inhalation and they are particularly resistant to macrophage and
neutrophil destruction.
Ref:-Pathology
by Philip Xiu
4th edition published by Elsevier
155.ANS.C
EXPLANTION:SILICOSIS
Free silica / silicon dioxide (Si02)
Nodular fibrosis (3- 4mm)
LP - months - 6 yrs
CXR - snow storm appearance
Notifiable disease (FA 1948)
Miner Act-1952
No effective treatment.
Fibroticchanges cannot be reversed.
First reported in India-Kolar gold mines.(1947)
The lesions consist of concentric whirled bundles of hyalinized collagen fibers with scattered black
pigment. The pattern is usually random, with some nodules located in perivascular or peribronchiolar
location, and others scattered within the lung parenchyma. The lesion in image 2.2 is viewed under
polarized light, and demonstrates numerous polarizable crystals. This is a characteristic picture of
silicosis.
The lesion forms in reaction to cellular damage thought to be caused by interaction between SiOH groups
on the hydrated surface of the silica crystals, the offending particles, with cellular macromolecules in
macrophages, including phospholipids and proteins. There is damage to lipid membranes, with cell injury
and death leading to release of a soluble protein factor that stimulates fibroblast proliferation and collagen
synthesis.
Ref:-http://library.aua.edu.ag/webpath/webpath/labs/pulm1lab/pulm1a02.htm
156.ANS.B
EXPLANTION:Continuous:
Height of children
Weight of cars
18
Time to wake up in the morning
Speed of the train
Ref:-http://www.usablestats.com/lessons/datatypes2
157.ANS.C
EXPLANTION:Nominal Variables
Nominal variables allow for only qualitative classification. That is, they can be measured only in terms of
whether the individual items belong to certain distinct categories, but we cannot quantify or even rank
order the categories: Nominal data has no order, and the assignment of numbers to categories is purely
arbitrary. Because of lack of order or equal intervals, one cannot perform arithmetic (+, -, /, *) or logical
operations (>, <, =) on the nominal data. Typical examples of such variables are:
Gender:
1. Male
2. Female
Marital Status:
1. Unmarried
2. Married
3. Divorcee
4. Widowe
Ref:-http://www.unesco.org/webworld/idams/advguide/Chapt1_3.htm
158.ANS.C
EXPLANTION:A pie chart is a circular chart divided into sectors, illustrating numerical proportion. In a pie chart, the arc
length of each sector (and consequently its central angle and area), is proportional to the quantity it
represents. While it is named for its resemblance to a pie which has been sliced, there are variations on
the way it can be presented. The earliest known pie chart is generally credited to William Playfair's
Statistical Breviary of 1801
An obvious flaw exhibited by pie charts is that they cannot show more than a few values without
separating the visual encoding (the “slices”) from the data they represent (typically percentages). When
slices become too small, pie charts have to rely on colors, textures or arrows so the reader can
understand them. This makes them unsuitable for use with larger amounts of data. Pie charts also take
up a larger amount of space on the page compared to the more flexible alternative of bar charts, which do
not need to have separate legends, and can also display other values such as averages or targets at the
same time.
Ref:-http://en.wikipedia.org/wiki/Pie_chart#Use.2C_effectiveness_and_visual_perception
159.ANS.B
EXPLANTION:Statisticians and quality control technicians gather data to determine correlations (relationships) between
such events. Scatter plots will often show at a glance whether a relationship exists between two sets of
data. Remember when making a scatter plot, do NOT connect the dots.
Ref:-http://www.regentsprep.org/regents/math/algebra/ad4/scatter.htm
160.ANS.A
EXPLANTION:The five-number summary is a descriptive statistic that provides information about a set of observations.
It consists of the five most important sample percentiles:
the sample minimum (smallest observation)
the lower quartile or first quartile
19
the median (middle value)
the upper quartile or third quartile
the sample maximum (largest observation)
In order for these statistics to exist the observations must be from a univariate variable that can be
measured on an ordinal, interval or ratio scale.
The five-number summary provides a concise summary of the distribution of the observations. Reporting
five numbers avoids the need to decide on the most appropriate summary statistic. The five-number
summary gives information about the location (from the median), spread (from the quartiles) and range
(from the sample minimum and maximum) of the observations. Since it reports order statistics (rather
than, say, the mean) the five-number summary is appropriate for ordinal measurements, as well as
interval and ratio measurements.
It is possible to quickly compare several sets of observations by comparing their five-number summaries,
which can be represented graphically using a boxplot.
In addition to the points themselves, many L-estimators can be computed from the five-number summary,
including interquartile range, midhinge, range, mid-range, and trimean.
The five-number summary is sometimes represented as in the following table:
median
1st quartile
3rd quartile
Minimum
Maximum
Ref:-http://en.wikipedia.org/wiki/Five-number_summary
162.ANS.D
EXPLANTION:The patient has deep vein thrombosis (DVT). She requires anticoagulation with heparin given as an initial
intravenous bolus followed by a continuous infusion to maintain the partial thromboplastin time 2–3 times
the control, and continued until 24 hours after the international normalized ratio (INR) is elevated to 2–3
times normal as a result of warfarin.
Ref:-Rapid Review USMLE Step 3
by David D.K. Rolston and Craig Nielsen
1st edition published by Elsevier
162.ANS.C
EXPLANTION:Respiratory Acidosis - Compensation-he compensatory response is a rise in the bicarbonate level
This rise has an immediate component (due to a resetting of the physicochemical equilibrium point) which
raises the bicarbonate slightly.
Next is a slower component where a further rise in plasma bicarbonate due to enhanced renal retention
of bicarbonate. The additional effect on plasma bicarbonate of the renal retention is what converts an
"acute" respiratory acidsosis into a "chronic" respiratory acidosis.
As can be seen by inspection of the Henderson-Hasselbalch equation (below), an increased [HCO3-] will
counteract the effect (on the pH) of an increased pCO2 because it returns the value of the [HCO3]/0.03
pCO2 ratio towards normal.
pH = pKa + log([HCO3]/0.03 pCO2)
4.5.2 Buffering in Acute Respiratory Acidosis
The compensatory response to an acute respiratory acidosis is limited to buffering.
By the law of mass action, the increased arterial pCO2 causes a shift to the right in the following reaction:
20
CO2 + H2O <-> H2CO3 <-> H+ + HCO3In the blood, this reaction occurs rapidly inside red blood cells because of the presence of carbonic
anhydrase. The hydrogen ion produced is buffered by intracellular proteins and by phosphates.
Consequently, in the red cell, the buffering is mostly by haemoglobin. This buffering by removal of
hydrogen ion, pulls the reaction to the right resulting in an increased bicarbonate production. The
bicarbonate exchanges for chloride ion across the erythrocyte membrane and the plasma bicarbonate
level rises. In an acute acidosis, there is insufficient time for the kidneys to respond to the increased
arterial pCO2 so this is the only cause of the increased plasma bicarbonate in this early phase. The
increase in bicarbonate only partially returns the extracellular pH towards normal.
Empirically, the [HCO3-] rises by 1 mmol/l for every 10mmHg increase in pCO2 above its reference value
of 40mmHg. For example, if arterial pCO2 has risen acutely from 40mmHg to 60mmHg (due to decreased
alveolar ventilaton) then this acute rise of 2 tens (i.e. 60-40=20mmHg rise) results in a rise of plasma
bicarbonate by 2 from its reference value of 24mmol/l up to 26 mmol/l. Consequently, we would predict
that if this acute respiratory acidosis was the only base disorder present, then plasma bicarbonate would
be 26mmol/l.
Though very important for carriage of carbon dioxide in the blood, the bicarbonate system is not itself
responsible for any buffering of a respiratory acid-base disorder. This is because a system cannot buffer
itself. If HCO3 were to react with H+ produced from the dissociation of H2CO3 this would just produce
H2CO3 again - reversing the reaction is not 'buffering'.
Ninety-nine percent of the buffering of an acute respiratory acidosis occurs intracellularly. Proteins
(especially haemoglobin in red cells) and phosphates are the most important buffers involved. These take
up the H+ produced from the dissociation of H2CO3. This intracellular buffering results in a further
increase in intracellular [HCO3] because it pulls the CO2 hydration reaction to the right. The HCO3 that
leaves the cell causes the rise in extracellular HCO3. The amount of buffering is limited by the
concentration of protein as that is low relative to the amount of carbon dioxide requiring buffering.
In summary: Compensation for an acute respiratory acidosis is by intracellular buffering and plasma
bicarbonate rises slightly as a result of this buffering. The buffering is predominantly due to intracellular
proteins; the bicarbonate system does not contribute to this buffering.
4.5.3 Chronic Respiratory Acidosis: Renal Bicarbonate Retention
With continuation of the acidosis, the kidneys respond by retaining bicarbonate.
If the respiratory acidosis persists then the plasma bicarbonate rises to an even higher level because of
renal retention of bicarbonate.
Thus in a chronic respiratory acidosis there are TWO factors present which elevate the plasma
bicarbonate:Firstly: The acute physicochemical change and consequent buffering esp by intracellular protein.
(Immediate onset - as occurs with an acute respiratory acidosis.)
Secondly: The renal retention of bicarbonate as renal function is altered by the elevated arterial pCO2
and additional bicarbonate is added to the blood passing through the kidney. (Slow onset)
Studies have shown that an average 4 mmol/l increase in [HCO3-] occurs for every 10mmHg increase in
pCO2 from the reference value of 40mmHg. For example, if arterial pCO2 has risen from 40mmHg to
60mmHg (due to decreased alveolar ventilaton) and remained elevated for several days, then this chronic
rise of "2 tens" (i.e. 60-40=20mmHg rise = 2 rises of 10mmHg) results in a rise of plasma bicarbonate by
8 from its reference value of 24mmol/l up to 32 mmol/l. Consequently, we would predict that if this chronic
respiratory acidosis was the only base disorder present, then plasma bicarbonate would be 32mmol/l.
The renal response in underway by 6 to 12 hours with a maximal effect reached by 3 to 4 days. This
maximal effect is not sufficient to return plasma pH to normal, but because of the additional renal
contribution, the pH is returned towards normal much more than occurs in an acute respiratory acidosis.
The response occurs because increased arterial pCO2 increases intracellular pCO2 in proximal tubular
cells and this causes increased H+ secretion from the PCT cells into the tubular lumen. This results in:
increased HCO3 production which crosses the basolateral membrane and enters the circulation (so
plasma [HCO3] increases.)
increased Na+ reabsorption in exchange for H+ and less in exchange for Cl- (so plasma [Cl-] falls)
21
increased 'NH3' production to 'buffer' the H+ in the tubular lumen (so urinary excretion of NH4Cl
increases)
4.5.4 'Maximal compensation' versus 'full compensation'?.
The increase in plasma [HCO3] results in an increase in amount of bicarbonate filtered in the kidney and
this amount increases as plasma bicarbonate continues to increase. Eventually a new steady state is
reached which is referred to as ‘maximal compensation’.
This level of compensation has long been believed to be less than that required to return the plasma pH
to normal. That is the actual compensation ('maximal compensation') is less than 'full compensation'. If
the pH was found to actually be within the normal range, the interpretation of this was that there was a
co-existing metabolic alkalosis (e.g. due to use of diuretics or corticosteroids) or there had been transient
hyperventilation from the stress of arterial puncture.
A recent study1 examined the actual maximal response in a group of patients with stable chronic
hypercapnic respiratory failure without a clinical condition or medications those could cause a metabolic
alkalosis. The majority of these patients had pH values in the normal range as the compensation was
greater than that predicted by the classic 4 for 10 rule. They found that bicarbonate increased by 5.1
mmols/l for every 10mmHg pCO2 rise.
Consequently, a diagnosis of mild metabolic alkalosis should not be made in patients with stable chronic
respiratory acidosis with pH values in the normal range unless there is other evidence (e.g. use of
thiazide or loop diuretics, or corticosteroids) consistent with the diagnosis.
In summary, the compensation for hypercapnia is:
Acute: Buffering only and predominantly intracellular (99%)
Chronic: Renal retention of bicarbonate (in addition to buffering)
Summary notes about the compensation terms
Maximal compensation refers to the actual maximal amount of compensation that is typically seen in a
patient with a simple acid-base disorder.
Full compensation refers to the amount of compensation that would correct the pH all the way back to
within the normal range.
The general rule for all acid-base disorders is that the body's compensatory response is almost never
sufficient to return the plasma pH to normal. If the pH is normal then it suggests that a second,
compensating acid-base disorder is present. Contrary to this 'classic' teaching, a recent paper1 suggests
that in many patients with chronic stable hypercapnia, compensation may be sufficient to return pH to
within the normal range.
Ref:-http://www.anaesthesiamcq.com/AcidBaseBook/ab4_5.php
163.ANS.B
EXPLANTION:A nonvolatile acid (also known as a fixed acid or metabolic acid) is an acid produced from sources other
than carbon dioxide, and is not excreted by the lungs. They are produced from e.g. an incomplete
metabolism of carbohydrates, fats, and proteins. All acids produced in the body are nonvolatile except
carbonic acid, which is the sole volatile acid. Common nonvolatile acids in humans are lactic acid,
phosphoric acid, sulfuric acid, acetoacetic acid, and beta-hydroxybutyric acid. Humans produce about 1 1.5 mmoles of H+ per kilogram per day.
The following reactions result in nonvolatile acids:
sulfur-containing amino acid oxidations
e.g. methionine or cysteine --> Urea + CO2 + H2SO4 --> 2H+ + SO42phosphorus-containing compound metabolism --> H3PO4 --> H+ + H2PO4cationic amino acid oxidation
e.g. lysine or arginine -->Urea + CO2 + H2O + H+
Non-metabolizable organic acid production:
22
HA --> H+ + AIncomplete metabolism of carbohydrates, fats, and proteins
--> e.g. lactic acid or keto-acids
Carbonic acid is the chemical compound with the formula H2CO3 (equivalently OC(OH)2). It is also a
name sometimes given to solutions of carbon dioxide in water (carbonated water), because such
solutions contain small amounts of H2CO3. In physiology, carbonic acid is described as volatile acid or
respiratory acid, because it is the only acid excreted as a gas by the lungs
Ref:-http://www.anaesthesiamcq.com/AcidBaseBook/ab4_5.php
164.ANS.B
EXPLANTION:A “loop” diuretic like furosemide would relieve the excessive preload and allow the cardiac output to
improve and the edema and ascites to subside.
Ref:-Cardiovascular Physiology
by Achilles J. Pappano and Withrow Gil Wier
10th edition published by Elsevie
165.ANS.B
EXPLANTION:The elevated left atrial pressure would be transmitted back to the wedged catheter (wedge pressure).
Ref:-Cardiovascular Physiology
by Achilles J. Pappano and Withrow Gil Wier
10th edition published by Elsevie
166.ANS.C
EXPLANTION:300 mL O2/min /[(18 − 8) mL O2/100 mL blood] = 3000 mL/min = 3 L/min.
Ref:-Cardiovascular Physiology
by Achilles J. Pappano and Withrow Gil Wier
10th edition published by Elsevie
167.ANS.D
EXPLANTION:Decreased alveolar ventilation, secondary to her obstructive lung disease, led to an increase in PCO2
because perfused regions of her lungs were not ventilated (ventilation–perfusion defect). In those poorly
ventilated regions of the lungs, CO2 could not be expired. The increase in PCO2 caused a decrease in
her arterial pH.
The HCO3− concentration is always increased to some extent in simple respiratory acidosis. The extent
of this increase depends on whether the disorder is acute or chronic. In acute respiratory acidosis, the
HCO3− concentration is modestly increased secondary to mass action effects that are explained by the
following reactions. As CO2 is retained and PCO2 increases, the reactions are driven to the right, causing
an increase in HCO3− concentration.
equation
In chronic respiratory acidosis, the increase in HCO3− concentration is much greater because, in addition
to mass action effects, the kidney increases the synthesis and reabsorption of “new” HCO3− (renal
compensation). This compensation for respiratory acidosis occurs in the intercalated cells of the late
distal tubule and collecting ducts, where H+ is secreted and new (i.e., newly synthesized) HCO3− is
reabsorbed. When arterial PCO2 is chronically elevated, renal intracellular PCO2 is elevated as well. This
increased intracellular PCO2 supplies more H+ for urinary secretion and more HCO3− for reabsorption
Why is this renal response, which causes an increase in the blood HCO3− concentration, called a
compensation? Compensation for what? The increase in HCO3− concentration is “compensating for,” or
bringing, the pH toward normal, as shown in the Henderson–Hasselbalch equation:
equation
23
In respiratory acidosis, CO2 (the denominator of the ratio) is increased secondary to hypoventilation. This
increase in PCO2 causes a decrease in arterial pH. In the chronic phase of respiratory acidosis, the
kidneys increase the HCO3− concentration (the numerator). This increase tends to normalize the ratio of
HCO3− to CO2 and the pH. Although Bernice had retained significant amounts of CO2 (her PCO2 was
69 mm Hg), her pH was only modestly acidic (7.32) 6 months prior to her death. Bernice “lived” at an
elevated PCO2 of 69 mm Hg because her kidneys compensated, or brought, her pH almost to normal.
(Incidentally, healthy persons “live” at a PCO2 of 40 mm Hg.)
For simple chronic respiratory acidosis, HCO3− is expected to increase by 0.4 mEq/L for every 1-mm Hg
increase in PCO2. To calculate the expected, or predicted, increase in HCO3−, we determine how much
the PCO2 was increased above the normal value of 40 mm Hg, then multiply this increase by 0.4. The
predicted change in HCO3− is added to the normal value of HCO3− to determine the predicted HCO3−
concentration.
Increase in PCO2
= 69 mm Hg − 40 mm Hg
= 29 mm Hg
Predicted increase in HCO3− = 29 mm Hg × 0.4 mEq/L per mm Hg
= 11.6 mEq/L
Predicted HCO3− concentration = 24 mEq/L + 11.6 mEq/L
= 35.6 mEq/L
In other words, if Bernice had simple chronic respiratory acidosis, her HCO3− concentration should have
been 35.6 mEq/L, based on the expected renal compensation. At the initial visit, her actual HCO3−
concentration was 34 mEq/L, which is very close to the predicted value. Therefore, we can conclude that
Bernice had only one acid–base disorder at the earlier visit: simple chronic respiratory acidosis.
Clinical findings in respiratory acidosis are related to the degree and duration of the respiratory acidosis
and whether hypoxemia is present. A precipitous rise in PCO2 can lead to confusion, anxiety, psychosis,
asterixis, seizures, and myoclonic jerks, with progressive depression of the sensorium and coma at an
arterial PCO2 greater than 60 mm Hg (CO2 narcosis). Hypercapnia, which increases cerebral blood flow
and volume, can lead to symptoms and signs of elevated intracranial pressure, including headaches and
papilledema. Other findings in acute respiratory acidosis include signs of catecholamine release, such as
skin flushing, diaphoresis, and increased cardiac contractility and output. Symptoms of chronic
hypercapnia include fatigue, lethargy, and confusion, in addition to the findings seen in acute
hypercapnia.
Ref:-Goldman’s Cecil Medicine
by Lee Goldman and Andrew I. Schafer
24th edition published by Elsevie
168.ANS.E
EXPLANTION:CLINICAL MANIFESTATIONS OF LUNG CANCER
Lung cancer is more common in men and most frequently presents in patients in their 50s and 60s. It is
uncommon in patients less than 30 years of age. The clinical manifestations of lung carcinoma result from
the following:
Local tumor growth
Intrathoracic metastases
Extrathoracic metastases
A paraneoplastic syndrome
Local Tumor Growth and Intrathoracic Metastases
Table 3-12 Symptoms in Patients with Lung Cancer
Symptoms associated with lung cancer are largely nonspecific and are present in a minority of patients
when the tumor is first detected radiographically (Table 3-12). Most patients with cancer are smokers and
have a history of chronic cough; any change in their pattern of cough or sputum production should be
considered significant.
24
Symptoms are most common in patients with central carcinomas involving large bronchi or mediastinal
structures, and in patients with tumors metastatic to hilar or mediastinal lymph nodes. Symptoms of
central carcinoma can include the following:
Bronchial obstruction with cough, hemoptysis, wheezing, dyspnea, or fever due to postobstructive
pneumonia
Hoarseness from involvement of the recurrent laryngeal nerve
SVC syndrome resulting from mediastinal invasion or metastases
Dysphagia from esophageal invasion or compression
Chylothorax from involvement of the thoracic duct
Diaphragmatic paralysis from involvement of the phrenic nerve
Peripheral lung cancers can be associated with pleural or chest wall invasion, resulting in chest pain,
dyspnea, or cough. At autopsy, about 10% of patients with lung cancer have chest wall involvement by
direct extension. Peripheral carcinomas in the superior sulcus can produce Pancoast syndrome.
Superior Vena Cava Syndrome
Obstruction or narrowing of mediastinal vessels as a result of mediastinal invasion or lymph node
metastases is a relatively common manifestation of lung cancer. Because of its location and relatively thin
wall, the SVC is particularly susceptible to involvement by tumor. SVC obstruction is associated with the
SVC syndrome. About 65% to 85% of cases of SVC syndrome are caused by lung carcinoma. Symptoms
and signs of SVC syndrome include the following:
Facial fullness, flushing, and cyanosis
Headache
Edema of the upper extremities
Prominent veins on the face and upper chest
Other common causes of SVC syndrome include granulomatous diseases such as histoplasmosis or
tuberculosis involving the mediastinum (granulomatous mediastinitis), and venous thrombosis.
Extrathoracic Metastases
Small cell carcinoma grows rapidly and tends to metastasize early. Adenocarcinoma may grow slowly but
metastasizes early. Squamous cell carcinoma may grow rapidly but tends to metastasize late.
Hematogenous spread to many sites has been reported with lung cancer, but the central nervous system,
bones, liver, and adrenal glands are most commonly involved. Such metastases preclude successful
surgical resection. The use of imaging studies to detect distant metastases in lung cancer patients is
discussed below.
Paraneoplastic Syndromes
Paraneoplastic syndromes are disorders associated with malignant neoplasms but not directly related to
the physical effects of the primary tumor. Such syndromes are present in 10% of patients with lung
carcinoma (20% of those with small cell carcinoma) and result from the production of hormones or
peptides by the tumor, antigen–antibody interactions resulting from tumor products, or neurovascular
mechanisms. They may precede pulmonary findings by months or even years. A large variety of
manifestations have been reported.
Hypertrophic Pulmonary Osteoarthropathy
Digital clubbing and hypertrophic pulmonary osteoarthropathy (HPO) are the most common cutaneous
disorders. Eighty percent of cases of HPO in adults are due to lung cancer. Squamous cell carcinoma is
most commonly associated with HPO; HPO is uncommon with small cell carcinoma. Relief of symptoms
typically follows resection of the primary neoplasm. Pulmonary osteoarthropathy may precede discovery
of the lung neoplasm by up to 2 years.
Vascular Disorders
Thrombophlebitis has an increased incidence in lung cancer patients and is most common with
adenocarcinoma.
Endocrine Disorders
Cushing’s syndrome, resulting from tumor secretion of ectopic adrenocorticotrophic hormone (ACTH),
consists of weakness, hyperglycemia, polyuria, and hypokalemic alkalosis; in patients with lung cancer it
is typically of rapid onset and progression. It can be associated with any cell type of tumor. Cushing’s
syndrome is most common in patients with carcinoid tumor, and approximately 30% of patients with an
ectopic cause of this syndrome have a bronchial carcinoid tumor. Small cell carcinoma is associated with
25
Cushing’s syndrome in less than 5% of cases, at least partially because of the short life expectancy of
patients with this tumor.
Hypercalcemia associated with lung cancer is most common with squamous cell carcinoma. Occasionally
it is associated with bone metastases, but it is more often due to production of a peptide similar to
parathyroid hormone. Other mediators such as prostaglandin have also been implicated in
hypercalcemia.
Inappropriate antidiuretic hormone secretion, resulting in hyponatremia, is usually associated with small
cell carcinoma. Although 50% of patients with small cell carcinoma have elevated levels of antidiuretic
hormone, only 10% to 15% have hyponatremia, and less than 5% of patients have symptoms attributable
to this syndrome.
Neuromuscular Syndromes
Neuromuscular syndromes associated with lung cancer may result from immunologic mechanisms. Small
cell carcinoma is most commonly responsible. Symptoms may precede the diagnosis of the tumor or may
be the first sign of recurrence.
Eaton-Lambert syndrome is characterized by proximal muscle weakness similar to myasthenia gravis,
with the exception that muscle strength increases (rather than decreases) with use. Hyporeflexia and
autonomic dysfunction are also parts of this syndrome. Small cell carcinoma is the most common
malignant tumor associated with Eaton-Lambert syndrome; it can also be seen with extrathoracic tumors,
and approximately 50% of cases are not associated with a detectable tumor. This syndrome apparently
results from the production of anti–calcium channel antibodies, which impairs the release of acetylcholine.
Peripheral neuropathy is associated with small cell carcinoma and less often with squamous cell
carcinoma and adenocarcinoma. Antineuronal nuclear antibodies are likely involved. Chronic intestinal
pseudoobstruction, limbic encephalitis, necrotizing myelopathy, and visual paraneoplastic syndrome also
occur with small cell carcinoma and are associated with antineuronal nuclear antibodies. Symptoms from
the neuropathy may precede the discovery of the carcinoma by years; however, in most cases, advanced
disease is present.
Other neuromuscular manifestations include subacute cerebellar degeneration (ataxia, vertigo,
uncoordination) and dementia.
Ref:-Thoracic Imaging: Pulmonary and Cardiovascular Radiology
by W. Richard Webb and Charles B. Higgins
2nd edition published by Wolters Kluwer
169.ANS.C
EXPLANTION:As with acute myeloid leukemia, patients with acute lymphoblastic leukemia require induction
chemotherapy, which usually includes vincristine, doxorubicin and prednisone. The presence of the
chromosomal marker t9,22 in a patient with acute lymphoblastic leukemia carries a poor prognosis. The
only potential for cure in these patients is allogeneic bone marrow transplant.
Ref:-Rapid Review USMLE Step 3
by David D.K. Rolston and Craig Nielsen
1st edition published by Elsevier
170.ANS.A
EXPLANTION:Definitions
Acidemia → pH <7.36, alkalemia → pH >7.44
Acidosis → process that increases [H+]; alkalosis → process that decreases [H+]
Primary disorders: metabolic acidosis or alkalosis, respiratory acidosis or alkalosis
Compensation
respiratory: hyper- or hypoventilation alters PaCO2 to counteract 1° metabolic process
renal: excretion/retention of H+/HCO3 to counteract 1° respiratory process
respiratory compensation occurs in minutes; renal compensation takes hours to days
compensation never fully corrects pH; if pH normal, consider mixed disorder
Determine primary disorder: ✓ pH, PaCO2, HCO3
26
Determine if degree of compensation is appropriate
Mixed disorders (more than one primary disorder at the same time)
If compensation less or greater than predicted, may be 2 disorders:
PaCO2 too low → concomitant 1° resp. alk.
PaCO2 too high → concomitant 1° resp. acid.
HCO3 too low → concomitant 1° met. acid.
HCO3 too high → concomitant 1° met. alk.
Normal pH but…
↑ PaCO2 + ↑ HCO3 → resp. acid. + met. alk.
↓ PaCO2 + ↓ HCO3 → resp. alk. + met. acid.
normal PaCO2 & HCO3, but ↑ AG → AG met. acid. + met. alk.
normal PaCO2, HCO3, & AG → no disturbance or non-AG met. acid. + met. alk.
Cannot have resp. acid. (hypoventilation) and resp. alk. (hyperventilation) simultaneously
ABG vs. VBG: concordant for pH (~0.04), HCO3 (~2 mEq) but not PaCO2 (~8±17 mmHg)
VBG can be used to screen for hypercarbia w/ PaCO2 cutoff ≥45 mmHg (100% Se),
but does not accurately assess degree of hypercarbia
METABOLIC ACIDOSIS
Initial workup (Nat Rev Nephol 2010;6:274)
✓ anion gap (AG) = Na+ – (Cl− + HCO3−) = unmeasured anions − unmeasured cations
if ↑ glc, use measured not corrected Na
expected AG is [albumin] × 2.5 (ie, 10 if albumin is 4 g/dL, 7.5 if albumin is 3 g/dL)
↑ AG → ↑ unmeasured anions such as organic acids, phosphates, sulfates
↓ AG → ↓ alb or ↑ unmeasured cations (Ca, Mg, K, Li, bromine, immunoglobulin)
If ↑ AG, ✓ delta-delta (ΔΔ = ΔAG/ΔHCO3) to assess if there is an additional metabolic acid-base
disturbance; ΔAG = (calculated AG – expected AG), ΔHCO3 = (24 – HCO3)
ΔΔ = 1–2 → pure AG metabolic acidosis
ΔΔ < 1 → AG metabolic acidosis and simultaneous non-AG acidosis
ΔΔ > 2 → AG metabolic acidosis and simultaneous metabolic alkalosis
Workup for AG metabolic acidosis
✓ for ketonuria (dipstick acetoacetate) or plasma β-hydroxybutyrate (βOHB)
nb, urine acetoacetate often not present in early ketoacidosis due to shunting to βOHB; ∴ acetoacetate
may later turn ⊕, but does not signify worsening disease
If ⊖ ketones, ✓ renal function, lactate, toxin screen, and osmolal gap
Osmolal gap (OG) = measured osmoles – calculated osmoles
calculated osmoles = (2 × Na) + (glucose / 18) + (BUN / 2.8) (can + [EtOH/4.6] if have EtOH level and
want to test if other ingestions)
OG >10 → suggests ingestion (see below)
for methanol/ethylene glycol: early on, OG precedes AG; later OG may be nl with ⊕ AG
Workup for non-AG metabolic acidosis (CJASN 2012;7:671)
Evaluate history for causes (see above)
✓ urine anion gap (UAG) = (UNa + UK) – UCl
UAG = unmeasured anions – unmeasured cations; as NH4+ is primary unmeasured cation, UAG is
indirect assay for renal NH4+ excretion (NEJM 1988;318:594)
⊖ UAG →↑ renal NH4+ excretion → appropriate renal response to acidemia
Ddx: GI causes, proximal RTA, ingestions or dilutional
⊕ UAG → failure of kidneys to secrete NH4+
Ddx: distal or hypoaldo RTA, early renal failure
nb, plasma K usually ↓ in distal and ↑ in hypoaldo RTA
UAG evaluation assumes Pt volume replete (UNa >25) & no AG met. acid. (which causes ⊕ UAG due to
excretion of organic anions)
Renal tubular acidoses (RTAs) (JASN 2002;13:2160; Int J Clin Pract 2011;65:350)
Proximal (Type II): ↓ proximal reabsorption of HCO3
1° (Fanconi’s syndrome = ↓ proximal reabsorption of HCO3, PO4, glc, amino acids), paraprotein (multiple
myeloma, amyloidosis), meds (acetazolamide, heavy metals, ifosfamide), renal transplant, ↓ Vit D, NRTIs
27
Distal (Type I): defective distal H+ secretion
1°, autoimmune (Sjögren’s, RA), nephrocalcinosis, meds (ampho, Li, ifosfamide); normally a/w ↓ K; if with
↑ K → sickle cell, obstruction, SLE, renal transplant
Hypoaldo (Type IV): ↑ K → ↓ NH3 synthesis/delivery → ↓ urine acid carrying capacity
↓ renin: diabetic nephropathy, NSAIDs, chronic interstitial nephritis, HIV
normal renin, ↓ aldo synthesis: 1° adrenal disorders, ACEI, ARBs, heparin
↓ response to aldosterone
meds: K-sparing diuretics, TMP-SMX, pentamidine, calcineurin inhibitors
tubulointerstitial disease: sickle cell, SLE, amyloid, diabetes
Combined (Type III): rarely discussed or clinically relevant, also called juvenile RTA, has distal & proximal
features, can be due to carbonic anhydrase II deficiency
Ref:-Pocket Medicine
by Marc S. Sabatine
5th edition published by Wolters Kluwer
171.ANS.E
EXPLANTION:Metastatic Tumors
Metastases to the Lungs: Introduction
Up to 50% of extrapulmonary malignancies will have lung metastases at autopsy, and in a quarter of
cases the lungs will be the only site of such metastases. This is due to several unique features of the
lungs, which make them ideal “seed and soil” territory, accepting most of the extrapulmonary metastases.
First, lungs receive the entire right-sided cardiac output continuously, with every heartbeat. Second, lungs
have the densest capillary network in the body, which consists of delicate, easily penetrable membranes.
Third, pulmonary vascular plexus receives most of the upstream lymph flow draining into the venous
system. In most instances lungs are the first organs exposed to drainage of tumor cell–bearing fluids,
such as lymph and blood.
Virtually any malignancy may spread to the lungs, but carcinomas (adenocarcinomas in particular) are far
more common than sarcomas, lymphomas, and melanomas simply because of their overall frequency.
The most common primary tumor sites are breast, colon, stomach, pancreas, kidney, melanoma,
prostate, liver, thyroid, adrenals, and male and female genital systems.
The most common route of metastatic tumor spreading to lungs is hematogenous. There are two principal
ways that tumor cells reach the lungs: by direct invasion into the veins or by lymphatic spreading, that is,
lymph drainage eventually entering the venous blood. Hematogenous metastases are most frequently
seen as bilateral nodules of variable size within middle and lower peripheral lung fields. These nodules in
comparison to primary tumors are more evenly contoured, less than 3 cm in diameter, more peripherally
located, and grow faster.
Lymphangitic spreading, also called lymphogenous spreading or lymphangitic carcinomatosis, is another
common route of metastases, especially from breast, stomach, bronchus, and Kaposi sarcoma. The most
common mechanisms of lymphangitic spreading are direct lymphatic invasion, spreading from adjacent
blood vessels, and retrograde lymphangitic spreading.
Direct extension as surface spreading, through chest wall, mediastinum, and diaphragm or via
permeating lymphatics, is also considered to be a mechanism of metastatic disease as long as tumor
deposits are discontinuous from the primary site.
Endobronchial spreading is the least common type but is important in the differential diagnosis of primary
tumors. The mechanisms of endobronchial spreading include hematogenous and lymphatic seeding,
spreading from adjacent lung parenchyma, mediastinum, and tumor nodules, or by aerogenous routes.
These types of metastases are commonly associated with primary carcinomas of breast, colon, kidney,
lung, rectum, melanoma, cervix, and sarcomas.
In addition to main routes, the secondary mechanisms of lung metastases include intrathoracic nodal
spreading, pleural metastases, and bronchopleural fistulas.
Ref:-Thoracic Pathology
by Aliya N. Husain
1st edition published by Elsevier
28
172.ANS.C
EXPLANTION:c is correct because the PR interval, which indicates the time for conduction from atria through the AV
node and ventricular conducting system to the ventricle, is normal.
Ref:-Cardiovascular Physiology
by Achilles J. Pappano and Withrow Gil Wier
10th edition published by Elsevie
173.ANS.A
EXPLANTION:Endobronchial Metastases
Endobronchial metastases are considered in a separate category because of their distinctive clinical
findings, principally the syndrome of “adult-onset asthma.” The reported incidence of endobronchial and
endotracheal metastatic disease is 1% to 18% of patients who also have intrapulmonary metastases. The
most common sites of tumor origin in patients from North America and Western Europe are the breast,
bone, soft tissue, large intestine, kidney, and skin (melanoma). More than one third of endobronchial
metastases are sarcomatous. In populations with a high prevalence of acquired immune deficiency
syndrome, the most common secondary malignancies of the bronchi are Kaposi sarcoma and malignant
lymphoma.59 Nasopharyngeal and laryngeal carcinomas are frequent sources of endobronchial
metastasis in Asia. Endobronchial metastases may be either hematogenous or lymphogenous.
Aerogenous spread of an upper-airway malignancy has also been suggested as a possibility. Tumors that
originate in the lung, hilar lymph nodes, or mediastinum may spread by direct extension into the bronchial
system. Endobronchial lesions cause symptoms early in their course of growth, namely, cough with
sputum production, dyspnea, wheezing, infection, and hemoptysis. However, up to 25% of patients are
asymptomatic. Radiographically, an endobronchial mass is typically visible only on computed tomography
or magnetic resonance scan; plain film studies commonly show only postobstructive consolidation or
atelectasis. The mean interval between diagnosis of the original tumor and the appearance of
endobronchial metastasis is 4 to 5 years. These patients have a poor survival, with a median of 11
months,patients with breast cancer may have a better prognosis.
Ref:-Thoracic Imaging: Pulmonary and Cardiovascular Radiology
by W. Richard Webb and Charles B. Higgins
2nd edition published by Wolters Kluwer
174.ANS.C
EXPLANTION:GENERAL PRINCIPLES
Pleural effusions are common in primary lung cancer and mesothelioma, as well as in advanced breast
cancer and lymphoma. Effusions may be malignant (positive fluid cytology or pleural biopsy) or
paramalignant (caused by indirect tumor effects). This distinction is critical in lung cancer since patients
with malignant effusions are not considered for potentially curative therapies (classified as stage IV
disease).
DIAGNOSIS
Routine thoracentesis with cytologic evaluation is typically adequate. However, effusions with negative
cytology should be further evaluated with thoracoscopy or open pleural biopsy (see Chapter 10,
Pulmonary Diseases).
TREATMENT
Prompt and complete drainage by therapeutic thoracentesis is necessary to avoid chronic fibrosis and
trapped lung. Observation for rate of reaccumulation after initial drainage is appropriate for most patients.
Rapidly reaccumulating effusions (<1 month) should be treated aggressively.
Pleurodesis (obliteration of the pleural space by fibrosis) by complete drainage and instillation of a
sclerosant (usually talc) will prevent recurrence of most malignant effusions. Requires hospitalization and
chest tube placement causing significant temporary pain.
29
Placement of an indwelling pleural catheter for intermittent outpatient drainage is an alternative. Pleural
fibrosis and resolution of the effusion occurs over several weeks in most patients.
Medical therapy may be sufficient in breast cancer or lymphoma but is otherwise ineffective.
Radiation of the pleural space is not feasible, but treatment of central masses will often alleviate
paramalignant effusions.
Ref:-Thoracic Imaging: Pulmonary and Cardiovascular Radiology
by W. Richard Webb and Charles B. Higgins
2nd edition published by Wolters Kluwer
175.ANS.E
EXPLANTION:Lung carcinoma commonly spreads via the lymphatic system. Diffuse involvement of the pulmonary
lymphatics results in the appearance of lymphangitic spread of carcinoma, often in association with hilar
node enlargement and pleura effusion. The classic plain film appearance is a unilateral or asymmetrical
increase in pulmonary interstitial markings, which may be associated with Kerley’s B lines; however,
atypical appearances are common. High-resolution CT typically shows interlobular septal thickening
hickening of the peribronchovascular interstitium, and thickening of fissures.
Lymph Node Enlargement
Hilar and mediastinal node enlargement is detected radiographically in up to 35% of lung cancers at
diagnosis, although 50% have evidence of node metastasis at surgery. The diagnosis of hilar and
mediastinal mass and lymph node enlargement is discussed in detail in other chapters.
Hilar enlargement visible on radiographs can reflect the primary tumor arising in a central location (usually
this results in a poorly marginated hilar mass that may be large; or metastases to hilar lymph nodes from
a peripheral lung primary (usually a well-defined hilar mass; Hilar mass is common in squamous cell
carcinoma. Enlargement of hilar lymph nodes in association with a central mass is characteristic of small
cell carcinoma
Hilar enlargement is the first detectable radiographic finding in 10% to 15% of cases of lung cancer.
Airway abnormalities (narrowing or obstruction) are common, but not invariably, seen in patients with a
hilar mass or lymph node enlargement.
Mediastinal Mass or Lymph Node Enlargement
Mediastinal lymph node metastases are common in patients with lung carcinoma, occurring in up to 40%
of patients at diagnosis, depending on the size, location, and cell type of the primary tumor. In patients
with a small lung nodule as the only presenting finding, mediastinal metastases are found in
approximately 20%. CT is most accurate in making this diagnosis.
Mediastinal lymph node enlargement is an uncommon plain film abnormality at initial presentation
When visible radiographically, lymph node enlargement is usually limited to the middle mediastinum and
is associated with a visible lung or hilar mass. The most common sites of mediastinal node enlargement
seen on plain radiographs are the right paratracheal mediastinum for right-sided tumors and the
aorticopulmonary window for left-sided tumors. The subcarinal lymph nodes are also commonly involved
but are difficult to recognize on plain radiographs unless they are quite large. It is unusual for mediastinal
lymph nodes to be involved radiographically without involvement of the hilum.
In some patients with lung carcinoma, a mediastinal mass may be the first and only presenting
abnormality, occurring in the absence of a visible lung mass. Isolated mediastinal masses are usually
associated with small cell carcinoma or poorly differentiated carcinoma.
Pleural Abnormalities
Pleural Effusions or Masses
Small pleural effusions are common in patients with lung cancer; pleural effusion occurs in 5% to 15% of
patients. They can result from pleural metastases, lymphatic obstruction in the hilum or mediastinum, or
inflammatory lung disease associated with bronchial obstruction. The term malignant effusion should be
reserved for effusions containing malignant cells. The presence of a pleural effusion, particularly when
bloody, indicates a poor prognosis in lung cancer, but only a malignant effusion rules out surgical
treatment.
Ref:-Thoracic Imaging: Pulmonary and Cardiovascular Radiology
by W. Richard Webb and Charles B. Higgins
30
2nd edition published by Wolters Kluwer
176.ANS.A
EXPLANTION:Carcinoma of the female breast
This is the most common malignancy in the female. About 42 000 women and 300 men are diagnosed
annually in the UK with carcinoma of the breast. Of the women diagnosed with breast cancer, 72% live
for more than 10 years. The incidence of the disease appears to be rising. The cause remains unknown.
Survival rate has improved significantly over the last 10–15 years. This probably reflects screening,
awareness and earlier diagnosis, use of tamoxifen and better chemotherapeutic agents.
Risk factors
•Family history – first-degree relatives with early-onset disease
•Previous breast carcinoma (recurrence versus metachronous disease)
•Atypical hyperplasia on previous biopsy
•Nulliparous women
•Early menarche
•Age (the older the more likely).
Symptoms and signs
Symptoms include: painless lump in the breast; nipple retraction or discharge; skin dimpling; breast
asymmetry; erythema over skin or nipple; symptoms of metastases, e.g. bone pain, headache,
breathlessness, jaundice. Examination may reveal: hard irregular mass, fixed to skin or fixed deeply;
erythema; Paget’s disease of nipple; peau d’orange; axillary glands mobile, fixed or matted;
supraclavicular nodes palpable; signs of metastases – liver (jaundice, hepatomegaly, ascites), lung or
pleural metastases (pleural effusion, consolidation), bone secondaries (bone tenderness or pathological
fractures), brain secondaries (headache, fits, personality change, papilloedema).
Investigations
•Triple assessment (examination + imaging + pathology)
•Metastases screen – either (A) CXR, liver ultrasound, bone scan, LFTs and calcium, FBC or (B) CT
thorax/abdomen, bone scan, LFTs and calcium, FBC. Frozen section is rarely performed nowadays as it
is 35% inaccurate.
•CT scan of brain if symptoms, but not routinely performed.
Clinical staging
Two forms are in wide use
Clinical staging
Two forms are in wide use
TNM Classification
Primary (T)
Tis – carcinoma in situ
T0 – no primary tumour located
T1 – tumour <2 cm
T2 – tumour 2–5 cm
T3 – tumour >5 cm
T4 – extension to chest wall
Nodes (N)
N0 – no nodal involvement
N1 – mobile ipsilateral axillary nodes
N2 – fixed ipsilateral axillary nodes
N3 – ipsilateral supraclavicular nodes
Metastases (M) M0 – no metastases
M1 – distant metastases
Treatment
Surgery to the breast ranges from wide local excision (WLE) to mastectomy + oncoplastic or
reconstructive options.
Important studies starting in the 1970s demonstrated that WLE with adjuvant radiotherapy produced
comparable local recurrence and disease-free survival rates to mastectomy alone. WLE therefore
became an option for women with smaller tumours in relatively larger breasts, i.e. where clearance is
31
possible while still conserving the breast. Larger tumours in smaller breasts may necessitate mastectomy
to achieve local control.
Adjuvant therapies include chemotherapy, anti-oestrogens (hormone) therapy and targeted therapies,
e.g. trastuzumab (Herceptin) – a monoclonal antibody targeted against HER2 protein, bisphosphonates.
Treatment options should be discussed with the patient. Preoperative counselling by the surgeon and a
specially trained breast care nurse should explain the treatment options and prepare the patient for
treatment. Recent trends are to more conservative management of breast cancer.
Alternative treatment modalities include neo-adjuvant chemotherapy (prior to surgery) which may
‘downstage’ a tumour to enable breast conservation in a patient with locally advanced disease that would
otherwise require a mastectomy. Also, primary endocrine therapy for oestrogen receptor positive cancers
can be offered as an alternative to surgery in patients who are at high risk of an anaesthetic, refuse
surgery, or are assessed to be at a higher risk of mortality from another cause.
Surgery for ductal carcinoma in situ, Tis (DCIS)
Forming 25–30% of screening-detected tumours, ductal carcinoma in situ represents an area of dysplasia
reaching, but not invading the basement membrane. It can be low, intermediate or high nuclear grade
and has a variety of histological subtypes. It represents a point in the evolution between atypia and
malignancy. Microcalcification may be present on mammography and it may occupy a small area of the
breast or be widespread. Core biopsies positive for DCIS, may not be a true reflection of the mass which
may contain areas of invasion. Surgery with clear histological margins is the primary aim which, in
widespread DCIS may require mastectomy. If histology confirms invasion then the axilla will require
staging (see below), and adjuvant therapy is offered accordingly.
Surgery for early breast cancer (T1, T2)
•Wide local excision: removal of the lump with a margin of normal breast tissue. If the lesion is impalpable
such as those detected by screening, a wire is passed to the tumour localized under ultrasound or
mammographic guidance, offering the surgeon a guide to locate the tumour. Specimens are oriented with
marker sutures and clips and an X-ray taken to assess clearance of margins. Surgery is now routinely
performed with a view to preserving the volume and contour of the breast (oncoplastic technique).
—
Neighbouring breast tissue can be mobilized to fill the resulting defect (tissue displacement), whereas
larger defects can be replaced with nearby tissue, e.g. latissimus dorsi muscle (tissue replacement)
sometimes after a delay to confirm complete excision.
—
Fat injection harvested by liposuction from the abdomen or thighs is becoming increasingly popular in
addressing partial defects.
—
In larger breasts, the wide local excision can be performed as part of a bilateral breast reduction
(therapeutic mammaplasty).
•Simple mastectomy either with immediate or delayed reconstruction. Most common mastectomy
operation is modified Patey procedure, i.e. pectoralis minor is preserved (Auchincloss mastectomy).
•Whichever excision technique is used, the nodal status of the axilla should be staged formally, either by
sentinel node biopsy, axillary node sampling or axillary clearance according to local protocol. Patients
with positive nodes on sampling would normally proceed to formal clearance of all the axillary lymph
nodes or radiotherapy. Devices enabling the intraoperative assessment of sentinel nodes/node samples
for evidence of cancer spread are currently being introduced.
Patients with early breast cancer should be considered for systemic adjuvant treatment, i.e.
chemotherapy and/or anti-oestrogen therapy. Anti-oestrogen therapy should only be used in those who
are oestrogen receptor +ve. Oestrogen blockade (tamoxifen) or oestrogen deprivation therapy with
aromatase inhibitors (anastrozole, letrozole, exemestane) may be used according to local protocol.
Ovarian oestrogen production can be stopped with oophorectomy, radiotherapy or goserelin injections.
Primary tamoxifen therapy without surgery may be used in elderly/unfit patients.
Surgery for advanced breast cancer (T3, T4)
•Locally advanced disease (no systemic disease) – neoadjuvant therapy –
chemotherapy/radiotherapy/surgery
•Salvage mastectomy
Palliation (stage IV)
32
•Local palliation (e.g. radiotherapy for fungating lesions)
•Radiotherapy to localized bony metastases
•Aspiration of pleural effusions and instillation of cytotoxic agents
•Hormonal manipulation, e.g. tamoxifen
•Chemotherapy.
Extensive surgery may be required for chest wall defects requiring grafting with myocutaneous flaps, e.g.
latissimus dorsi flaps.
Recurrent disease
Local recurrent disease may be treated by radiotherapy if this has not been given to the area before.
Systemic disease may be treated by hormonal manipulation or chemotherapy.
Follow-up
•Routine self-examination of operation site and other breast
•Follow-up is in accordance with local protocols but should be continued for 5 years – patients who are
disease free then should be discharged
•Mammography to contralateral breast every other year and to ipsilateral breast, if conserved, every year.
Prognosis
•Stage I: 80% 5-year survival
•Stage II: 50% 5-year survival
•Stage III: 15% 5-year survival
•Stage IV: 5% 5-year survival.
Complications of mastectomy
Wound seromas, stiffness of the shoulder. Lymphoedema of the arm is a complication of axillary node
clearance. The main problems are psychological and should be managed both preoperatively and
postoperatively by expert counselling and support.
Breast reconstruction
Following mastectomy, many patients are well rehabilitated with an external prosthesis that fits in a bra.
Others, however, wish breast reconstruction. This may be performed immediately at the time of the initial
surgery or at a later date. Reconstruction after mastectomy can be achieved using implant-based or
autologous tissue according to patient choice, size of breast, fitness for surgery. Autologous transfer
imports healthy tissue via pedicled or microsurgical free tissue transfer which makes it a superior choice if
the chest wall has been irradiated.
Tissue can be recruited from:
•The back – pedicled latissimus dorsi muscle with overlying skin and fat based on the subscapular
vessels.
•The abdomen – skin and fat with the underlying rectus abdominis muscle based on the superior
epigastric vessels can be rotated into the mastectomy defect about this vascular pedicle (pedicled
transverse rectus abdominis musculocutaneous flap – TRAM). The free microsurgical TRAM is based on
the deep inferior epigastric vessels and a variant of this, the deep inferior epigastric perforator flap (DIEP)
spares the rectus abdominis muscle.
•The buttocks – skin and fat can be harvested based on perforators from the superior or inferior gluteal
vessels (SGAP or IGAP)
•The inner thigh – The free transverse upper gracilis (TUG) flap. A paddle of skin and fat is elevated with
the underlying gracilis muscle based on the medial circumflex femoral vessels.
Breast screening
This is based on the premise that early detection of breast cancer improves the prognosis. Current
screening in the UK invites women in the 50–70 age group to have two-view mammography every 3
years. By 2012, this age group will be extended to 47–73. Of the screened women, 5% are recalled for
further investigation. Suspicious lesions on mammography are biopsied by image-guided core biopsy or
mammotome biopsy – >90% preoperative diagnosis is expected by this technique. If screening biopsy
does not produce a result concordant with the imaging findings, then open biopsy is required: in
impalpable lesions this may be performed using a wire-localization technique (see above). If malignant,
appropriate treatment is undertaken.
Ref:-Thoracic Imaging: Pulmonary and Cardiovascular Radiology
by W. Richard Webb and Charles B. Higgins
2nd edition published by Wolters Kluwer
33
177.ANS.C
EXPLANTION:The pulse is totally irregular.
Ref:-Cardiovascular Physiology
by Achilles J. Pappano and Withrow Gil Wier
10th edition published by Elsevie
178.ANS.D
EXPLANTION:d is correct because there is greater activation of repolarizing K channels when heart rate increases.
Ref:-Cardiovascular Physiology
by Achilles J. Pappano and Withrow Gil Wier
10th edition published by Elsevie
179.ANS.E
EXPLANTION:In metabolic acidosis, the primary change is a fall in serum bicarbonate. The compensatory response is
to increase ventilation to reduce PCO2. Worsening acidosis elicits increasing alveolar ventilation.
Primary metabolic acidosis results from an imbalance between net acid production and net acid excretion
(NAE) in the form of urinary ammonium and acid phosphate. Consider the following relationship, where
Ux represents the urinary concentration and V̇ the urinary flow rate:
In a normal steady-state condition, the rate of excretion of net acid must be equal to the rate of
production. The normal production rate depends on diet. If net acid production is normal, metabolic
acidosis could occur because of a failure to reabsorb bicarbonate or a failure to elaborate enough urinary
buffers, as is the case in renal failure and renal tubular acidosis. An inequality also could develop if net
acid production were excessive or if large extrarenal bicarbonate losses were unable to be matched by
maximal adaptive increases in net acid excretion. Endogenous sources of acid include ketoacidosis and
lactic acidosis, whereas exogenous sources are metabolic products of ingested ethylene glycol or
methanol. On occasion, strong inorganic acids may be ingested. When net acid is retained in body fluids,
the serum bicarbonate concentration falls. However, maintenance of a constant serum HCO3−
concentration does not guarantee that there is a new steady state in which net acid production is equal to
net acid excretion because body buffers such as carbonate salts of bone may become depleted by
relentless acid retention, as in renal failure and distal renal tubular acidosis.
The causes of metabolic acidosis are usually categorized according to the presence of either a normal or
an elevated serum anion gap. The serum anion gap is the net charge difference when the sum of chloride
and bicarbonate is subtracted from the serum sodium concentration.
The normal anion gap is due to the unmeasured anionic charge associated predominantly with albumin.
When acidemia is present, albumin is in a more protonated form, which lowers the normal gap. In
alkalemia, the effect of pH is to increase the gap attributed to albumin. Each 1 g/dL of albumin contributes
approximately 2.8 mEq/L to the normal anion gap. The anion gap may be low with hypoalbuminemia or
with an increase in unmeasured cations, such as immunoglobulin G myeloma paraproteins, calcium,
lithium, or magnesium. When the anion gap is increased above the normal value of approximately 10 to
12 mEq/L by a nonchloride acid anion, an anion gap metabolic acidosis exists. The accompanying proton
is responsible for lowering the serum bicarbonate concentration. The degree of increase in the anion gap,
sometimes referred to as the gap delta, may be estimated by the difference between the observed anion
gap and a normal value of 10 to 12 mEq/L. A similar calculation for a change in serum HCO3− can be
made by subtracting the observed HCO3− from the normal value of about 25 mEq/L. Comparison of the
two values (the delta-delta) may help identify more complicated acid-base disorders. If the increase in the
anion gap is larger than the decrease in serum HCO3−, a process is raising the HCO3− level. The patient
may have a coexisting metabolic alkalosis or be compensating for chronic respiratory acidosis. If the
34
decreases in serum HCO3− are larger than the increases in the anion gap, a sign that another process is
lowering the HCO3− level, the patient may have an additional hyperchloremic acidosis or respiratory
alkalosis.
Ref:-Goldman’s Cecil Medicine
by Lee Goldman and Andrew I. Schafer
24th edition published by Elsevie
180.ANS.C
EXPLANTION:The first ABG shows normal acid–base status and significant hypoxia on air. The second ABG shows a
metabolic acidosis, and the patient is still very hypoxic despite receiving 40% O2. The PaO2 should have
increased but it has hardly changed at all, so pulmonary gas exchange has probably worsened in the
second ABG.
The metabolic acidosis is easy to diagnose without being told the HCO3-. PaCO2 has decreased but the
patient is acidaemic. These H+ ions must have come from somewhere but they have not come from CO2.
Thus, there must be a metabolic acidosis.
More on Acid–base disturbance (p. 443)
Ref:-davidsons-principles-practice-medicine-colledge-21st/chapter-8
181.ANS.B
EXPLANTION:Noradrenaline (norepinephrine) acts primarily in the peripheral vasculature, causing intense
vasoconstriction, with relatively little direct effect on the heart. Its major clinical use is in hyperdynamic
sepsis, when there is low systemic vascular resistance and blood pressure despite a high cardiac output.
In these circumstances vasoconstriction is desirable to restore blood pressure, even at the expense of a
small reduction in cardiac output.
Dobutamine and dopexamine are sometimes called ‘ino-dilators’, since they increase cardiac contractility
whilst simultaneously causing systemic vasodilatation. Sodium nitroprusside and GTN are both nitric
oxide donors that cause profound systemic vasodilatation.
More on Circulatory effects of commonly used vasoactive drug infusions (p. 194)
Ref:-davidsons-principles-practice-medicine-colledge-21st/chapter-8
182.ANS.E
EXPLANTION:Calcium channel blockers are used to control ventricular rate in patients with atrial fibrillation. They are,
however, not the drugs of choice.
Ref:-Rapid Review USMLE Step 3
by David D.K. Rolston and Craig Nielsen
1st edition published by Elsevier
183.ANS.C
EXPLANTION:Rheumatic fever is endemic in parts of Asia, Africa, and South America. The pulmonary and tricuspid
valves are rarely involved. The loud P2 is indicative of associated pulmonary hypertension.
Ref:-Rapid Review USMLE Step 3
by David D.K. Rolston and Craig Nielsen
1st edition published by Elsevier
184.ANS.D
EXPLANTION:-
35
Statins can cause myopathy and affect liver function. It is therefore recommended that creatinine kinase
and transaminase levels are checked before initiating treatment and periodically thereafter.
Ref:-Rapid Review USMLE Step 3
by David D.K. Rolston and Craig Nielsen
1st edition published by Elsevier
185.ANS.E
EXPLANTION:Both rosiglitazone and pioglitazone can cause fluid retention and in susceptible patients precipitate
congestive heart failure.
Ref:-Rapid Review USMLE Step 3
by David D.K. Rolston and Craig Nielsen
1st edition published by Elsevier
186.ANS.C
EXPLANTION:These long episodes of palpitations warrant further evaluation. Because the palpitations are episodic,
Holter monitoring or an Event monitor are the two methods that can be used to identify the arrhythmia.
Ref:-Rapid Review USMLE Step 3
by David D.K. Rolston and Craig Nielsen
1st edition published by Elsevier
187.ANS.E
EXPLANTION:This patient is in extremis and at high risk of imminent cardiac arrest because of hypoxia. Her normal
bicarbonate shows that this is an acute not a chronic deterioration, and therefore probably reversible. Her
oxygen must not be reduced, but unfortunately increasing it is unlikely to be enough to save her,
particularly as this may further decrease her respiratory drive, leading to a further increase in PaCO2 and
worsening respiratory acidosis. The best solution is mechanical ventilatory support, which can be
delivered by non-invasive bi-level ventilation or by intubation in an ITU.
More on Assessment and management of type II respiratory failure (p. 661)
Ref:-davidsons-principles-practice-medicine-colledge-21st/chapter-19
188.ANS.C
EXPLANTION:The patient’s history and medical background suggests claudication pain initially; critical limb ischemia is
suggested because the pain has become constant. This requires immediate intervention.
Ref:-Rapid Review USMLE Step 3
by David D.K. Rolston and Craig Nielsen
1st edition published by Elsevier
189.ANS.A
EXPLANTION:Spike-and-wave is the term that describes a particular pattern of the electroencephalogram (EEG)
typically observed during epileptic seizures. A spike-and-wave discharge is a regular, symmetrical,
generalized EEG pattern seen particularly during absence epilepsy, also known as ‘petit mal’ epilepsy.
The basic mechanisms underlying these patterns are complex and involve part of the cerebral cortex, the
thalamocortical network, and intrinsic neuronal mechanisms. The first spike-and-wave pattern was
recorded in the early twentieth century by Hans Berger. Many aspects of the pattern are still being
researched and discovered, and still many aspects are uncertain. The spike-and-wave pattern is most
commonly researched in absence epilepsy, but is common in several epilepsies such as Lennox-Gastaut
syndrome (LGS) and Ohtahara syndrome. Anti-epileptic drugs (AEDs) are commonly prescribed to treat
epileptic seizures, and new ones are being discovered with less adverse effects. Today, most of the
36
research is focused on the origin of the generalized bilateral spike-and-wave discharge. One proposal
suggests that a thalamocortical (TC) loop is involved in the initiation spike-and-wave oscillations.
Although there are several theories, the use of animal models has provided new insight on spike-andwave discharge in humans.
The EEG is the most important test in making a diagnosis of juvenile myoclonic epilepsy. The EEG in
untreated individuals is typically abnormal with what is known as a 3-6 Hz generalized polyspike and
wave discharge.
MRIs are typically normal. 30% of patients may show a photoparoxysmal response or an abnormal EEG
in response to flickering lights. It is important when someone has a question of juvenile myoclonic
epilepsy that if a normal EEG is obtained, that an EEG capturing sleep and wakefulness is performed in
order to make certain that the diagnosis has been accurately assessed.
What does juvenile myoclonic epilepsy look like?
The hallmark characteristics of juvenile myoclonic epilepsy are the presence of myoclonic jerks that occur
on awakening from sleep either in the morning or from a nap. They are typically described as shock-like,
irregular and arrhythmic movements of both arms. Sometimes these movements are restricted only to the
fingers making the patient or individual look clumsy or prone to dropping things.
In almost 20% of individuals, these myoclonic jerks occur primarily on one side as opposed to both sides
of the body.
Some types of myoclonus are normal and do not imply epilepsy. The most common of these is so-called
hypnic jerks, which most of us have just as we are falling asleep.
Generalized tonic-clonic seizures are reported in nearly all patients and absence seizures are seen in 1
out of 3 people with JME. A small percentage of patients may never develop a generalized tonic-clonic
seizure.
When the patient develops myoclonic jerks, they typically occur in clusters and may be the warning sign
prior to the generalized tonic-clonic seizure.
The most concerning aspect of juvenile myoclonic epilepsy is the presence of a condition known as
myoclonic status epilepticus. This occurs primarily when an individual awakens and has multiple
myoclonic seizures that do not readily stop. It is precipitated by sleep deprivation or missing medications.
All seizures, especially the myoclonic jerks, occur within 30 minutes to an hour of awakening. Myoclonic
jerks rarely occur at any other time unless the individual is quite tired.
The diagnosis of medial temporal lobe epilepsy is still by listening to a person describe their seizures or
hearing observations of a witness.
An MRI of the brain is considered the standard radiology procedure to see the characteristic
abnormalities associated with medial temporal lobe epilepsy.
An EEG is also essential - they often present with anterior temporal spike or sharp waves, which can
occur in both wakefulness and/or sleep.
Sometimes recording seizures in a video EEG monitoring unit is needed. This is often done to localize
seizures and determine if surgery could be helpful.
Ref:-http://en.wikipedia.org/wiki/Spike-and-wave, http://www.epilepsy.com/learn/types-epilepsysyndromes/temporal-lobe-epilepsy
190.ANS.B
EXPLANTION:Epilepsy can be a delayed consequence of head trauma. In fact, about 5% of all cases of epilepsy are
due to this.
Head trauma is very common in today's world. In addition to trauma from vehicle crashes and sporting
accidents, head injury is becoming the signature injury of modern warfare. Over 1 million Americans
sustain head trauma each year, but fortunately only a minority of these are severe. So how often does
civilian head trauma lead to epilepsy? It generally depends upon how severe the head trauma is. Mild
head trauma, with loss of consciousness for less than 30 minutes, is associated with barely increased risk
of developing epilepsy compared to the general population.
Severe head trauma can be defined as either loss of consciousness or amnesia for greater than a day or
internal bleeding in or around the brain. Severe head trauma leads to epilepsy in about 15% of adults and
37
about 30% of children. Injuries with actual penetration of the brain, like a bullet wound, are even more
likely to cause epilepsy, about to 25 to 50% of the time.
Studies have looked at whether treatment with seizure medicines immediately after trauma, before a
seizures occur, prevents epilepsy, the condition of spontaneously recurrent seizures. Unfortunately, it
does not. Medication simply suppresses seizures while the patient takes them. If a person does have a
seizure post injury, a clinician will place it in one of two categories: early seizures, in the first week after
an accident, or late seizures occurring more than a week after trauma. Only late seizures are considered
to be epilepsy. Early seizures are a risk factor for later epilepsy, but most of the time they pass
uneventfully. An early seizure may not require treatment, but a seizure or two occurring later would be
treated by many doctors with the usual anti-seizure medications.
Posttraumatic seizures may not appear for as long as 20 years after an accident. Laboratory studies
suggest that this may be due to the long-term repair process after head injury. This repair process
happens as new connections and circuits are formed in the brain over a period of years. While this aids
recovery of strength, speech and memory, it may also form hyper-excitable circuits that are prone to
seizures.
Head trauma can cause many problems in addition to seizures. People often develop migraine
headaches, memory and concentration problems, dizziness, mood swings and various other symptoms
known as the concussion syndrome. Even after seemingly minor trauma, these symptoms can persist for
months. With more severe head trauma, neurological symptoms sometimes can be permanent.
In conclusion, head trauma is common and usually mild, but severe head trauma can lead to epilepsy.
There is a great need for a long-term medication that will prevent the development of epilepsy after
trauma and other types of head injury, and research is currently being conducted to find one.
Ref:-Kaufman’s Clinical Neurology for Psychiatrists
by David Myland Kaufman and Mark J. Milstein
7th edition published by Elsevier
191.ANS.B
EXPLANTION:Idiopathic intracranial hypertension (IIH), previously known as pseudotumor cerebri, is characterized by
elevated intracranial pressure in the absence of any structural cause or CSF abnormality. The underlying
cause is not well understood. Before puberty, girls and boys are equally likely to be affected.
After puberty, there is a female predominance. Recent use of certain medications, including
corticosteroids, has been associated with IIH. Being overweight is also associated,
and weight loss is an important component of long-term management in overweight patients.
Elevated intracranial pressure should be suspected in a patient who complains of a headache that is
worse when laying flat; the patient may describe this as worse when they lay down to go to sleep, or that
the headache is waking them up from sleep. A fundoscopic examination is essential to evaluate for
papilledema, which is swelling of the optic nerve head due to increased pressure. However, early on this
finding can be absent. Neuroimaging, including venous imaging, is necessary to rule out other causes of
elevated intracranial pressure such as an intracranial mass or venous sinus thrombosis. Following
neuroimaging, lumbar puncture with measurement of opening pressure should be performed with the
patient in the lateral decubitus position with the legs extended. IIH can be vision threatening and close
monitoring of visual function is needed. Carbonic anhydrase inhibitors, such as acetazolamide,
are first-line therapies as they decrease CSF production. Propranolol and cyproheptadine are used as
migraine preventives in children; ibuprofen and sumatriptan are acute therapies for migraine.
Ref:-Rudolf Pediatrics 22 edn (Page 2177, Section 29: Disorders of the Nervous System,
Chapter 553: Hydrocephalus and Pseudotumor Cerebri)
192.ANS.C
EXPLANTION:-
38
Primary headache disorders are relatively common in pediatrics. This child meets diagnostic criteria for
migraine. Migraineurs typically describe a pounding or pulsating quality to their head pain; children may
describe this as “booming” like their heartbeat. Pediatric migraine can be shorter in duration than adult
migraine, with a minimum duration of 1 hour. In addition, symptoms such as photophobia (light sensitivity)
and phonophobia (sound sensitivity) can be inferred from the child’s behavior, such as pulling the covers
over their head. There is usually a family history of migraine, although it is not uncommon for the family
member to think it is “normal” to get frequent headaches. Interviewing family members directly
about the features of their headaches can be helpful. Tension-type headaches are not accompanied by
migrainousn features such as photophobia or nausea. Idiopathic intracranial hypertension is
characterized by head pain that is worse when lying flat, whereas migraineurs typically prefer to lay
down when they have severe headache. Cluster headache is a rare headache disorder that is
characterized by unilateral headache associated with at least 1 cranial autonomic symptom
(eg, ptosis, conjunctival injection, nasal congestion). While headache can occur postictally, isolated ictal
headache would be quite rare.
Ref:-Rudolf Pediatrics 22 edn,(Page 2217, Section 29: Disorders of the Nervous System,
Chapter 565: Migraine and Headache Disorders)
193.ANS.B
EXPLANTION:HSV infection may be acquired in utero, during the birth process, or during the neonatal period.
Intrauterine and postpartum infections are well described but occur infrequently. Postpartum transmission
may be from the mother or another adult with a nongenital (typically HSV-1) infection such as herpes
labialis. Most cases of neonatal herpes result from maternal infection and transmission, usually during
passage through a contaminated infected birth canal of a mother with asymptomatic genital herpes.
Transmission is well documented in infants delivered by cesarean section. Fewer than 30% of mothers of
an infant with neonatal herpes have a history of genital herpes. The risk for infection is higher in infants
born to mothers with primary genital infection (>30%) than with recurrent genital infection (<2%). Use of
scalp electrodes may also increase risk. (See Chapter 244, page 1101.)
Ref:-nelson-pediatrics-kliegman-behrman-19th
194.ANS.A
EXPLANTION:Sumatriptan nasal spray has been well studied in children for the treatment of acute migraine in children
as young as age 6. It is an appropriate choice in children who have severe head pain, particularly those
who have difficulty taking oral medication due to nausea and vomiting. Triptans are migrainespecific
therapies that treat head pain as well as associated migraine symptoms such as photophobia,
phonophobia, nausea, and vomiting. Her attack frequency is low enough that using sumatriptan would not
be expected to lead to medication overuse headache. Ibuprofen is an excellent first-line agent for many
children with acute migraine. However, in this patient’s case she cannot tolerate it due to emesis.
Ondansetron oral dissolving tablets could be used to treat the nausea, but will have no impact
on the headache. Oxygen therapy can be useful in cluster headache, another headache disorder, but is
not a proven therapy for migraine. Amitriptyline is a preventive migraine agent and not useful in the acute
setting.
Ref:-Rudolf Pediatrics 22 edn,(Page 2218, Section 29: Disorders of the Nervous System,
Chapter 565: Migraine and Headache Disorders)
195.ANS.C
EXPLANTION:This child suffers from a cluster headache, a primary headache disorder. Cluster headache pain is severe
in intensity, lateralized, and associated with at least 1 cranial autonomic symptom. Examples of cranial
autonomic symptoms include conjunctival injection, lacrimation, periorbital edema, and nasal congestion
or rhinorrhea. The cranial autonomic symptoms lateralize to the side of the headache. High-flow
39
oxygen via non-rebreather mask for 20 minutes is an effective acute therapy in some patients. Both
prochlorperazine (a dopamine receptor antagonist) and ketorolac (an NSAID) are used in the acute
treatment of migraine in children in the emergency room setting, with prochlorperazine being superior.
Topiramate is migraine preventive medication. Sodium valproate is also a preventive migraine
medication, although there are several small studies suggesting some efficacy of an IV load of sodium
valproate in the treatment of acute migraine.
Ref:-Rudolf Pediatrics 22 edn
196.ANS.C
EXPLANTION:Current Diagnosis
•Lung abscess is usually a complication of aspiration in patients with gingivitis or periodontal disease.
•Mixed aerobic-anaerobic copathogens are common, but sputum cultures are not reliable.
•Lung abscesses are rare in edentulous patients and should prompt evaluation for bronchial obstruction.
•CT scan of the chest is very useful in the diagnostic evaluation.
Current Therapy
•Prolonged antimicrobial therapy (6–8 weeks) and postural drainage are the cornerstones of therapy.
•Clindamycin (Cleocin) or alternative anaerobic coverage is required even if sputum cultures grow only
aerobic bacteria (colonizers or copathogens).
•Percutaneous or bronchoscopic drainage is reserved for selected cases refractory to medical
management or at a high risk for complications.
•Surgical resection is seldom needed because of the success of medical therapy.
•Evaluation and management of underlying predisposing conditions should not be neglected.
Definition and Classification
Lung abscess is defined as a focal area of necrosis of the lung parenchyma resulting from microbial
infection and usually measuring more than 2 cm in diameter. Smaller or multiple areas of necrosis in
contiguous areas are referred to as necrotizing pneumonia. Primary lung abscess (80%) is due to direct
infection or aspiration. Secondary lung abscess (20%) is secondary to bronchial obstruction,
immunodeficiency, pulmonary infarction, trauma, or complications from surgery. Lung abscesses can also
be classified according to pathogen (e.g., mixed anaerobic, Pseudomonas, mycobacterial, fungal) or
duration of symptoms (e.g., chronic with symptoms for more than a month before presentation).
Epidemiology and Risk Factors
Most lung abscesses result from aspiration of oral secretions in patients who harbor high bacterial
concentrations in the gingival crevices. Periodontal disease, especially gingivitis, is a major predisposing
condition, particularly in hosts impaired by altered sensorium due to alcoholism, anesthesia, coma, drug
overdose, seizures, or stroke. Because edentulous persons rarely develop lung abscesses, other causes
such as malignancy should be carefully sought. Patients with dysphagia, esophageal disease, poor
airway protection, or weak cough and respiratory clearance mechanisms are also at risk for developing
lung abscess.
Patients whose immune systems are compromised by malignancy, HIV infection, malnutrition, diabetes,
chronic use of corticosteroids, or previous organ transplantation are more likely to be infected with
aerobic bacteria, mycobacterial or fungal pathogens. These patients are more likely to have multiple
abscesses, less response to treatment, and a worse prognosis.
In children, consider secondary causes including foreign body aspiration, congenital cystic adenomatoid
malformation, pulmonary sequestration, cystic fibrosis, bronchiectasis, bronchogenic cyst, congenital
immunodeficiency, or severe underlying neurologic abnormality.
Lemierre’s disease or jugular vein suppurative thrombophlebitis, usually caused by Fusobacterium
necrophorum, is a rare infection that begins in the pharynx as a tonsillar or peritonsillar abscess and
spreads to involve the internal jugular vein, with septic emboli to the lung with secondary cavitations.
Pathophysiology
The development of a lung abscess usually starts when an insult (e.g., inoculum of highly contaminated
oral secretions) overcomes the pulmonary mechanisms of defense and begins a process of pneumonitis
in the dependent areas affected by aspiration. Depending on the microbiology and the intensity of the
40
inflammatory response, the acute pneumonitis evolves to tissue necrosis after 7 to 14 days and
subsequent cavitation. At first, the enclosing wall is poorly defined, but with time and progressive fibrosis
it becomes more discrete. When a communication with the airway exists, the suppurative debris from the
abscess can partially drain, leaving an air-containing cavity with a radiographic air-fluid level.
Occasionally, abscesses rupture into the pleural cavity yielding an empyema or a bronchopleural fistula .
Primary abscesses due to aspiration are much more common on the right side than the left and are most
often single. The most common locations include the superior segments of the lower lobes and the
posterior segment of the right upper lobe.
Clinical Manifestations
Most patients present with insidious symptoms that evolve over a period of weeks to months. Cough
productive of copious amounts of putrid, foul-smelling sputum that occurs in paroxysms after changing
position are characteristic. Fevers, chills, night sweats, chest pain, dyspnea, general malaise, and fatigue
are common. Hemoptysis can vary from blood-streaked sputum to life-threatening hemorrhage. Physical
findings can include fever, tachycardia, periodontal disease, halitosis, signs of lung consolidation or
pleural effusion, amphoric breath sounds, and occasionally clubbing of the fingers and toes can appear
within a few weeks after the onset of an abscess.
Diagnosis
Lung abscess is easily diagnosed when there is a classic clinical presentation with indolent symptoms
lasting more than 2 weeks in a host with predisposing risk factors and a chest radiograph revealing a
cavitary infiltrate or an air-fluid level. However, numerous pathogens are associated with this syndrome,
and attempts to establish microbiological diagnosis and exclude other conditions are warranted.
Computed tomography (CT) scans can be useful for better anatomic definition, to evaluate possible
associated conditions such as malignancy, and to rule out pleural involvement. Distinguishing between a
lung abscess and an empyema with an associated bronchopleural fistula leading to an air-fluid level can
sometimes be challenging, but it is crucial because the management of these conditions is very different.
Features that suggest empyema include a lenticular shape or a larger diameter of the air-fluid level on the
lateral view of the chest film, an obtuse angle of the cavity with the chest wall, and a split pleural sign with
contrast enhancement of the pleura.
Most lung abscesses are caused by anaerobic or mixed aerobic and anaerobic infections. Anaerobic
bacteria include Peptostreptococcus, Prevotella, Bacteroides spp., and Fusobacterium spp. and are
difficult to isolate owing to technical issues and contamination by upper airway flora. Other pathogens,
including Staphylococcus aureus, Klebsiella pneumonia, Pseudomonas, Burkholderia pseudomallei,
Nocardia, Actinomyces, and mycobacterial or fungal organisms, are more likely to occur in secondary
lung abscesses.
Sputum Gram stains and culture should be performed in all patients but interpreted with caution because
prior antimicrobial therapy can inhibit growth, and contaminant strains can be misleading. Even when
there is abundant growth of a species of aerobic bacteria, treatment should still be directed at covering
anaerobes.
In the absence of positive blood or pleural fluid cultures, microbiological confirmation of a lung abscess
requires other invasive methods such as a transthoracic needle aspirates (TTNA) or bronchoscopy with
bronchoalveolar lavage (BAL) or protected specimen brush (PSB). The best timing for bronchoscopy is
controversial because early intervention has the highest diagnostic yield but at the risk of provoking
spillage of a relatively contained abscess into additional lobes or the contralateral lung. In patients who
are edentulous or in whom there is a high suspicion for malignancy, the indication for bronchoscopic
evaluation is almost universal but should be scheduled when the risk for clinical deterioration (e.g.,
spillage with resultant respiratory failure) has been minimized.
Differential Diagnosis
In addition to the multiple necrotizing infections or an empyema with a bronchopleural fistula (see earlier),
there are many noninfectious diseases that can cause cavitary lung lesions and mimic a lung abscess.
The differential diagnosis includes neoplasm (primary or metastatic), bullae or cyst with air-fluid level,
bronchiectasis, necrotizing vasculitis, or pulmonary infarction. In patients with multiple cavitary lesions,
consider septic emboli.
Treatment
41
Lung abscess is best treated with a prolonged course of adequate antimicrobials and postural drainage.
Percutaneous or bronchoscopic drainage and surgery are considered only for selected patients whose
disease is refractory to standard care.
Initial empiric antibiotic treatment for a typical community-acquired lung abscess should consist of
intravenous clindamycin (Cleocin) 600 to 900 mg every 6 to 8 hours, which has been shown to be
superior to penicillin. For patients with a nosocomial or health care–associated lung abscess, additional
coverage for enteric gram-negative pathogens including Pseudomonas aeruginosa and Staphylococcus
aureus is appropriate.
Alternative antimicrobial options include ampicillin–sulbactam (Unasyn)1 1.5 to 3.0 g IV every 6 hours,
piperacillin–tazobactam (Zosyn)1 3.35 g IV every 6 hours, cefoxitin (Mefoxin) 2 to 3 g every 6 to 8 hours,
or a combination of moxifloxacin (Avelox)1 400 mg IV daily and metronidazole (Flagyl) 500 mg IV every 6
to 8 hours. The use of metronidazole as single agent has been associated with a high therapeutic failure
rate.
After defervescence and radiographic improvement, parenteral antibiotics can be switched to oral
bioequivalent therapy for 6 to 8 weeks or longer depending on the course. Shorter antimicrobial courses
are associated with a high rate of relapse. Most experts suggest continuing therapy until there is
radiographic resolution or a small stable lesion.
Indications for percutaneous or bronchoscopic drainage include persistent sepsis after 5 to 7 days of
antimicrobial therapy, abscesses larger than 4 cm that are under tension or enlarging, and need for
mechanical ventilator support. Percutaneous drainage should only be considered when there is a
reasonable abscess–pleura symphysis and no associated coagulopathy.
Postural drainage and chest physiotherapy facilitate removal of pus, relieving symptoms and improving
gas exchange. Surgical resection is required in less than 10% of patients whose disease is refractory to
medical management.
Finally, evaluation and management of the predisposing conditions leading to aspiration should take
place after the patient is stabilized. This can include swallowing assessment, dental work, or oral surgery.
Ref:-Conn’s Current Therapy 2013
by Edward T. Bope and Rick D. Kellerman
1st edition published by Elsevier
197.ANS.D
EXPLANTION:Respiratory Effects of Opioids
The respiratory depressant actions of opioids are their most serious adverse effect. Although significant
adverse events related to opioid-induced respiratory depression are presumably preventable, they persist
with a perioperative incidence of approximately 0.1% to 1%, no matter what the route of administration.
Even though some early studies indicated both μ- and δ-opioid receptor involvement, a recent report
demonstrated that activation of the μ-opioid receptor in the caudal medullary raphe region, which is
important for pain regulation and respiratory modulation, inhibits the ventilatory response to hypercapnia
in anesthetized rats.Furthermore, administration of morphine or M6G did not produce significant
respiratory depression in μ-opioid receptor knock-out mice.Polymorphism of the μ-opioid receptor at
nucleotide position 118, which is known to affect M6G-induced analgesia, does not significantly change
susceptibility to the respiratory depressive effect of M6G.This result may suggest that analgesia and
respiratory depression are mediated by different signal transduction mechanisms activated by the μopioid receptor. In dogs, naltrindole, a highly selective antagonist of the δ-opioid receptor, can reverse the
sufentanil-induced respiratory depression without significant effect on the analgesic action of sufentanil
Several studies have reported a depressant effect of morphine on respiratory mucus transport, which is
one of the most important defenses against respiratory tract infections. However, morphine had no effect
on the beating frequency of nasal cilia in vitro.
Therapeutic Effects
Opioids, by decreasing pain and central ventilatory drive, are effective agents in preventing
hyperventilation induced by pain or anxiety. The lack of adequate pain relief can also cause postoperative
respiratory dysfunction. Opioids can be used as postoperative analgesics to prevent respiratory
42
dysfunction. The antitussive actions of opioids are well known and central in origin. However, fentanyl,
sufentanil, and alfentanil curiously elicit a brief cough in up to 50% of patients when injected by
intravenous bolus. Fentanyl, administered via a peripheral intravenous cannula, provoked cough when
injected rapidly, but the incidence was decreased significantly as the injection time was increased or by
the administration of 1.5 mg/kg lidocaine 1 minute before fentanyl administration. In addition, current
smokers had a lower incidence of cough than nonsmokers did, whereas heavy smokers did not.
Opioids are also excellent agents for depressing upper airway, tracheal, and lower respiratory tract
reflexes, but the mechanism is not clear. Although opioids can affect the contractile responses of airway
smooth muscles, the clinical significance and relevance of opioid-induced effects on airway resistance
remain controversial.Opioids blunt or eliminate somatic and autonomic responses to tracheal intubation.
They allow patients to tolerate endotracheal tubes without coughing or “bucking.” Opioids can also help
avoid increases in bronchomotor tone in asthma. In addition, fentanyl has antimuscarinic,
antihistaminergic, and antiserotoninergic actions and may be more effective than morphine in patients
with asthma or other bronchospastic diseases.
Nontherapeutic Effects
Opioids activating the μ-receptor cause dose-dependent depression of respiration, primarily through a
direct action on brainstem respiratory centers. How the various respiratory centers involved in the
ventilatory drive, respiratory rhythm generation, chemoreception, and neural integration are affected by
opioids is unclear.
The stimulatory effect of CO2 on ventilation is significantly reduced by opioids. The hypercapneic
response can be separated into central and peripheral components. Morphine-induced changes in the
central component were equal in men and women, whereas changes in the peripheral component were
larger in women. In addition, the apneic threshold and resting end-tidal PCO2 are increased by opioids .
Opioids also decrease the hypoxic ventilatory drive.
The respiratory rate is usually drastically slowed in opioid overdose, although a hypoxic CNS insult can
counter this effect. The prolonged expiratory time in the respiratory cycle induced by opioids frequently
results in greater reductions in respiratory rate than in tidal volume. Monitoring of breath intervals can
sensitively detect fentanyl-induced respiratory depression and can be used as a measure of dynamic
opioid effect.High doses of opioids usually eliminate spontaneous respirations without necessarily
producing unconsciousness. Patients receiving high doses of opioids may still be responsive to verbal
command and often breathe when directed to do so.
The peak onset of respiratory depression after an analgesic dose of morphine is slower than after
comparable doses of fentanyl: 30 ± 15 minutes versus 5 to 10 minutes. The respiratory depression
induced by small doses of morphine usually lasts longer than after equipotent doses of fentanyl.
Sufentanil (0.1 to 0.4 μg/kg) produces shorter-lasting respiratory depression and longer-lasting analgesia
than fentanyl does (1.0 to 4.0 μg/kg). Plasma fentanyl concentrations of 1.5 to 3.0 ng/mL are generally
associated with significant decreases in CO2 responsiveness. With higher doses of fentanyl (50 to 100
μg/kg), respiratory depression can persist for many hours. When moderately large (20 to 50 μg/kg or
greater) doses of fentanyl are used, the potential need for postoperative mechanical ventilation should be
anticipated. The effects of remifentanil, no matter what the dose, are attenuated rapidly and completely
within 5 to 15 minutes after termination of its administration. In healthy humans, the median effective
concentration (EC50) causing depression of minute ventilation with remifentanil and alfentanil was 1.17
and 49.4 ng/mL, respectively.
Although the mechanism by which pain modulates ventilatory control is unknown, daily clinical practice
provides indirect evidence that pain stimulates ventilation, especially during emergence from anesthesia.
Combes and colleagues demonstrated that pain relief via a nerve block in knee surgery patients who had
previously been treated with morphine PCA increases the incidence of abnormal respiratory events
associated with oxygen desaturation.
Naloxone is standard therapy for opioid-induced respiratory depression. However, there have been
reports of naloxone-resistant respiratory depression after intrathecal morphine.
Older patients are more sensitive to the anesthetic and respiratory depressant effects of opioids. They
experience higher plasma concentrations of opioids when administered on a weight basis. Morphine can
produce greater respiratory depression on a weight basis in neonates than in adults because it easily
penetrates the brain in neonates and infants with incomplete blood-brain barriers.
43
The respiratory depressant effects of opioids are increased or prolonged (or both) when administered
with other CNS depressants, including the potent inhaled anesthetics, alcohol, barbiturates,
benzodiazepines, and most of the intravenous sedatives and hypnotics. Exceptions are droperidol,
scopolamine, and clonidine, which do not enhance the respiratory depressant effects of fentanyl or other
opioids.
Although opioid action is usually dissipated by redistribution and hepatic metabolism rather than by
urinary excretion, the adequacy of renal function may influence the duration of opioid activity. In renal
insufficiency, the respiratory depressant properties of the morphine metabolite M6G would become
evident as it accumulated. One study indicated that M6G is a somewhat weaker respiratory depressant
than morphine is.
Hypocapnic hyperventilation enhances and prolongs postoperative respiratory depression after fentanyl
(10 and 25 μg/kg). Intraoperative hypercapnia produces the opposite effects. Possible explanations for
these findings include increased brain opioid penetration (increased un-ionized fentanyl with hypocapnia)
and removal (decreased CBF with hypocapnia). In patients who hyperventilate because of anxiety or
pain, even small intravenous doses of opioids can result in transient apnea because of acute shifts in
apneic thresholds.
Delayed or recurring respiratory depression has been reported with most opioids. Mechanisms for this
phenomenon may include augmented release of fentanyl or other opioids from skeletal muscle into the
systemic circulation on rewarming, shivering, motion, or any other condition that enhances muscle
perfusion.
Ref:-Miller’s Anesthesia
by Ronald D. Miller
7th edition published by Elsevier
198.ANS.C
EXPLANTION:In spirometry
Three patterns of flow-volume abnormalities can be detected: (1) variable intrathoracic obstructions, (2)
variable extrathoracic obstructions, and (3) fixed upper airway obstructions. Reproducing these findings
on every effort is important because spurious nonreproducible reductions in inspiratory flow are not
uncommon after completion of forced expirations in subjects without upper airway obstruction. Examples
of variable intrathoracic obstruction include localized tumors of the lower trachea or mainstem bronchus,
tracheomalacia, and airway changes associated with polychondritis.
Variable upper airway obstructions demonstrate flow reductions that vary with the phase of forced
respirations. Variable intrathoracic obstructions demonstrate reduction of airflow during forced expirations
with preservation of a normal inspiratory flow configuration. This is observed as a plateau across a broad
volume range on the expired flow limb of the flow-volume curve. The reduction in airflow results from a
narrowing of the airway inside the thorax, in part because of a narrowing or collapse of the airway
secondary to extraluminal pressures exceeding intraluminal pressures during expiration.
Flow reduction must be consistent on every effort to be considered actual flow limitation. Fixed upper
airway obstruction may be caused by postintubation stenosis, goiter, endotracheal neoplasms, and
bronchial stenosis. Variable extrathoracic obstruction may be caused by bilateral and unilateral vocal cord
paralysis, vocal cord constriction, reduced pharyngeal cross-sectional area, and airway burns. Variable
intrathoracic obstruction may be caused by tracheomalacia, polychondritis, and tumors of the lower
trachea or main bronchus.
Flow reduction must be consistent on every effort to be considered actual flow limitation. Fixed upper
airway obstruction may be caused by postintubation stenosis, goiter, endotracheal neoplasms, and
bronchial stenosis. Variable intrathoracic obstruction may be caused by tracheomalacia, polychondritis,
and tumors of the lower trachea or main bronchus. Variable extrathoracic obstruction may be caused by
bilateral and unilateral vocal cord paralysis, vocal cord constriction, reduced pharyngeal cross-sectional
area, and airway burns.
Ref:-http://emedicine.medscape.com/article/303239-overview
199.ANS.D
44
EXPLANTION:Functional Residual Capacity (FRC) is the volume of air present in the lungs, specifically the parenchyma
tissues, at the end of passive expiration. At FRC, the elastic recoil forces of the lungs and chest wall are
equal but opposite and there is no exertion by the diaphragm or other respiratory muscles.
FRC is the sum of Expiratory Reserve Volume (ERV) and Residual Volume (RV) and measures
approximately 2400 mL in a 70 kg, average-sized male. It can not be estimated through spirometry, since
it includes the residual volume. In order to measure RV precisely, one would need to perform a test such
as nitrogen washout, helium dilution or body plethysmography.
A lowered or elevated FRC is often an indication of some form of respiratory disease. For instance, in
emphysema, the lungs are more compliant and therefore are more susceptible to the outward recoil
forces of the chest wall. Emphysema patients often have noticeably broader chests because they are
breathing at larger volumes. In healthy humans, FRC changes with body posture. Obese patients will
have a lower FRC in the supine position.
The helium dilution technique and pulmonary plethysmograph are two common ways of measuring the
functional residual capacity of the lungs.
Ref:-http://en.wikipedia.org/wiki/Functional_residual_capacity
200.ANS.D
EXPLANTION:Tracheomalacia (from trachea and the Greek μαλακία, softening) is a condition characterized by flaccidity
of the tracheal support cartilage which leads to tracheal collapse especially when increased airflow is
demanded.
The trachea normally dilates slightly during inspiration and narrows slightly during expiration. These
processes are exaggerated in tracheomalacia, leading to airway collapse on expiration. The usual
symptom of tracheomalacia is expiratory stridor or laryngeal crow.
If the condition extends further to the bronchi (if there is also bronchomalacia), it is termed
tracheobronchomalacia. The same condition can also affect the larynx, which is called laryngomalacia.
Type 1 — congenital, sometimes associated with tracheoesophageal fistula or esophageal atresia
Type 2 — extrinsic compression sometimes due to vascular rings
Type 3 — acquired due to chronic infection or prolonged intubation or inflammatory conditions like
relapsing polychondritis
Ref:-http://en.wikipedia.org/wiki/Tracheomalacia
201.ANS.C
EXPLANTION:Assessment of a child's height and weight is one of the best indicators of his or her general health and
well-being. Abnormal growth might indicate the existence of underlying disease in the apparently normal
child. Early detection and diagnosis of short stature minimizes the impact of any underlying health
condition and optimizes final adult height. However, short stature in children is frequently unrecognized in
early childhood and thus diagnosed at a late age, which decreases the opportunity to intervene and
improve both their health outcomes and stature. A child's actual height or length results from initial length
at birth and the rate (or velocity) of growth over time. Growth velocity is highest at birth and progressively
decreases until the pubertal growth spurt causes the adolescent increase in height with sudden
deceleration to a growth velocity of 0 as epiphyseal fusion occurs
Ref:-http://www.ncbi.nlm.nih.gov/pmc/articles/PMC3738943/
45
202.ANS.C
EXPLANTION:What Causes Primary Bedwetting?
The cause is likely due to one or a combination of the following:
The child cannot yet hold urine for the entire night.
The child does not waken when his or her bladder is full.
The child produces a large amount of urine during the evening and night hours.
The child has poor daytime toilet habits. Many children habitually ignore the urge to urinate and put off
urinating as long as they possibly can. Parents usually are familiar with the leg crossing, face straining,
squirming, squatting, and groin holding that children use to hold back urine.
What Causes Secondary Bedwetting?
Secondary bedwetting can be a sign of an underlying medical or emotional problem. The child with
secondary bedwetting is much more likely to have other symptoms, such as daytime wetting. Common
causes of secondary bedwetting include the following:
Urinary tract infection: The resulting bladder irritation can cause pain or irritation with urination, a stronger
urge to urinate (urgency), and frequent urination (frequency). Urinary tract infections in children may
indicate another problem, such as an anatomical abnormality.
Diabetes: People with diabetes have a high level of sugar in their blood. The body increases urine output
to try to get rid of the sugar. Having to urinate frequently is a common symptom of diabetes.
Structural or anatomical abnormality: An abnormality in the organs, muscles, or nerves involved in
urination can cause incontinence or other urinary problems that could show up as bedwetting.
Neurological problems: Abnormalities in the nervous system, or injury or disease of the nervous system,
can upset the delicate neurological balance that controls urination.
Emotional problems: A stressful home life, as in a home where the parents are in conflict, sometimes
causes children to wet the bed. Major changes, such as starting school, a new baby, or moving to a new
home, are other stresses that can also cause bedwetting. Children who are being physically or sexually
abused sometimes begin bedwetting.
Ref:-http://www.webmd.com/sleep-disorders/guide/bedwetting-causes?page=2#1
203.ANS.B
EXPLANTION:By weight, the average human adult male is approximately 65% water. However, there can be
considerable variation in body water percentage based on a number of factors like age, health, weight,
and gender. In a large study of adults of all ages and both sexes, the adult human body averaged ~65%
water. However, this varied substantially by age, sex, and adiposity (amount of fat in body composition).
The figure for water fraction by weight in this sample was found to be 48 ±6% for females and 58 ±8%
water for males.
The body water constitutes as much as 73% of the body weight of a newborn infant, whereas some
obese people are as little as 45% water by weight. These figures are statistical averages, and so like all
biostatistics, the estimation of body water will vary with factors such as type of population, age of people
sampled, number of people sampled, and methodology. So there is not, and cannot be, a figure that is
exactly the same for all people, for this or any other physiological measure
Ref:-http://en.wikipedia.org/wiki/Body_water#In_humans
204.ANS.A
EXPLANTION:Hydrocephalus is quite common in children with a myelomeningocele and must be evaluated with a head
ultrasound study or CT.
Ref:-(See Chapter 585 in Nelson Textbook
of Pediatrics, 17th ed.)
205.ANS.B
46
EXPLANTION:An EEG will help define the presence of a seizure focus and if there is a specific seizure type. A normal
EEG may help the pediatric neurologist determine if anticonvulsant therapy is needed.
Ref:-(See Chapter 586 in Nelson
Textbook of Pediatrics, 17th ed.)
206.ANS.B
EXPLANTION:Young age is a risk factor for hepatic and perhaps pancreatic complications of valproic acid. It is much
safer in 12-yr-old children than those younger than 2 yr.
Ref:-Nelson 17th edn
207.ANS.D
EXPLANTION:In addition, phenobarbital and carbamazepine are also associated with an acute drug hypersensitivity
syndrome involving skin, mucous membranes, and the liver.
Ref:-Nelson 17th edn
208.ANS.B
EXPLANTION:Macroorchidism is a genetic disorder found in males where a subject has abnormally large testes. The
condition is commonly inherited in connection with fragile X syndrome, which is also the second most
common genetic cause of mental disabilities. This contrasts with Microorchidism, which is the condition of
abnormally small testes Martin Bell syndrome (fragile X syndrome) is the most
common heritable form of mental retardation in males. It ischaracterized by large testes, long face,
prominent jaw, large prominant ears and steteotyped behaviour and speech.
Ref:-http://en.wikipedia.org/wiki/Macroorchidism
209.ANS.D
EXPLANTION:Wolman disease is an A.recessive condition due to the deficiency of acid lipase (acid esterase). It
presents with chronic diarrhea, vomiting, failure to thrive, and abdominal distension due to massive
hepatosplenomegaly. Calcification of the adrenal is a striking finding shown by USS or X-ray.
Prognosis is very poor and most of the infants die by 1 year.
Ref:210.ANS.C
EXPLANTION:Presentation of cystic fibrosis with a rash is rare, with only 19 previously reported cases. This unusual
presentation is associated with false negative sweat tests, delays in diagnosis and considerable mortality.
Multiple nutritional deficiencies, the aberrant production of prostaglandins and free-radical mediated
damage have been implicated in the pathogenesis of this kwashiorkor-like dermatitis. In spite of the rarity
of this presentation, recognition of the rash is important, not only to expedite the diagnosis, but also to
gain insight into the disease. We present a further case to highlight this unusual presentation and discuss
potential pathophysiological mechanisms.
Ref:-DOI:10.1016/j.jcf.2006.05.003
211.ANS.B
EXPLANTION:The articulation between the dorsal septum and the nasal bones in the midline
is referred to as the "keystone" area. The significance of that area is that sufficient stability
and fixation must be maintained or reconstituted at that area to prevent postoperative collapse
47
or settling of the dorsal septum.
Ref:-Bailey's Head & Neck Surgery Otolaryngology 2014 by Lippincott Williams PAGE 2980
212.ANS.B
EXPLANTION:Meniere disease is an absolute contraindication for stapedectomyfstapedotomy.
When the endolymphatic space is dilated ( endolymphatic hydrops) , the saccule may be
enlarged to the point that it adheres to the undersurface of the stapes footplate. A stapes
procedure can injure the saccule and result in profound sensorineural hearing loss. PAG E 2489
Ref:-Bailey's Head & Neck Surgery Otolaryngology 2014 by Lippincott Williams
213.ANS.B
EXPLANTION:Immediate onset facial paralysis (no voluntary movement) indicates an acute
severe injury. This suggests possible disruption of the nerve. Options A and D describe
residual function at onset and delayed onset paralysis, respectively. The Glasgow Coma
score has no predictive impact on facial nerve status or recovery. It addresses gross motor
function and response to pain.
Ref:-Bailey's Head & Neck Surgery Otolaryngology 2014 by Lippincott WilliamsPAG E 241 3
214.ANS.D
EXPLANTION:The physician's subj ective impression of the presence or absence of a sleep
disorder is inaccurate. The gold standard test for OSA is a sleep study. PAG E 2 1 52
Ref:-Bailey's Head & Neck Surgery Otolaryngology 2014 by Lippincott Williams
215.ANS.C
EXPLANTION:TE speech has been rated the most desirable form of alaryngeal speech by both
speech pathologists and patients, and is the preferred method of alaryngeal speech by
naive listeners . PAG E 1 987
Ref:-Bailey's Head & Neck Surgery Otolaryngology 2014 by Lippincott Williams
216.ANS.B
EXPLANTION:Surgical anaesthesia Extends from onset of regular respiration to cessation of spontaneous
breathing. This has been divided into 4 planes which may be distinguished as:
Plane 1 Roving eyeballs. This plane ends when eyes become fixed.
Plane 2 Loss of corneal and laryngeal reflexes.
Plane 3 Pupil starts dilating and light reflex is
lost.
Plane 4 Intercostal paralysis, shallow abdominal
respiration, dilated pupil.
As anaesthesia passes to deeper planes, progressively—muscle tone decreases, BP falls,
HR increases with weak pulse, respiration decreases in depth and later in frequency also—
thoracic lagging behind abdominal.
Ref:-Lippincott's Anesthesia - Sikka, Pau
217.ANS.C
EXPLANTION:IV. Medullary paralysis Cessation of breathing to failure of circulation and death. Pupil is
48
widely dilated, muscles are totally flabby, pulse is thready or imperceptible and BP is very low.
Many of the above indices have been robbed by the use of atropine (pupillary, heart rate), morphine
(respiration, pupillary), muscle relaxants (muscle tone, respiration, eye movements,reflexes) etc. and the
modern anaesthetist has to depend on several other observations to gauge the depth of anaesthesia.
• If eyelash reflex is present and patient is making swallowing movements—stage II has not been
reached.
• Loss of response to painful stimulus (e.g. pressure on the upper nasal border of orbit)
— stage III has been reached.
• Incision of the skin causes reflex increase in respiration, BP rise or other effects; insertion
of endotracheal tube is resisted and induces coughing, vomiting, laryngospasm; tears appear in eye;
passive inflation of lungs is resisted—anaesthesia is light.
• Fall in BP, cardiac and respiratory depression are signs of deep anaesthesia.
Ref:-Lippincott's Anesthesia - Sikka, Pau
218.ANS.D
EXPLANTION:Atracurium besylate is a neuromuscular-blocking drug or skeletal muscle relaxant in the category of nondepolarizing neuromuscular-blocking drugs, used adjunctively in anesthesia to facilitate endotracheal
intubation and to provide skeletal muscle relaxation during surgery or mechanical ventilation. Atracurium
is classified as an intermediate-duration non-depolarizing neuromuscular-blocking agent.
Atracurium is susceptible to degradation by Hofmann elimination and ester hydrolysis as components of
the in vivo metabolic processes The initial in vitro studies appeared to indicate a major role for ester
hydrolysis but, with accumulation of clinical data over time, the preponderence of evidence indicated that
Hofmann elimination at physiological pH is the major degradation pathway[7] vindicating the premise for
the design of atracurium to undergo an organ-independent metabolism
Ref:-http://en.wikipedia.org/wiki/Atracurium_besilate#Clinical_pharmacology
219.ANS.A
EXPLANTION:Tracheal Tube
The safest method of securing a tracheal tube remains debatable. In general, the technique used should
be the one the operator is most familiar with. The method is generally unimportant as long as the
(potential) cervical spine injury is recognised and reasonable care taken (4).
The ATLS recommends a nasotracheal tube in the spontaneously breathing patient, and orotracheal
intubation in the apnoeic patient. MANUAL in-line axial stabilization must be maintained throughout. The
hard collar may interfere with intubation efforts and the front part may be removed to facilitate intubation
as long as manual stabilisation is in effect.
Blind nasal intubation is successful in 90% of patients but requires multiple attempts in up to 90% of
these. Nasotracheal intubation is (relatively) contraindicated in patients with potential base of skull
fracture or unstable mid-face injuries. In addition, it may produce haemorrhage in the airway, making
other airway manipulations difficult or impossible. Nasotracheal intubation in non-trauma patients is often
accomplished by rotating or flexing the neck to align the tube correctly. This is not possible in the trauma
patient and the procedure becomes more difficult. In the spontaneously breathing patient however, one
can hear movement of air at the end of the tracheal tube and thus line the tube up with the trachea.
Orotracheal intubation is generally accepted as the more usual method for securing the airway in the
trauma patient. It is the fastest and surest method of intubating the trachea. At Shock Trauma in
Baltimore, Maryland (5) more than 3000 patients were intubated orally with a modified rapid sequence
induction technique with pre-oxygenation and cricoid pressure. Ten percent of these patients were found
to have cervical spine injury and none deteriorated neurologically following intubation.
49
Atlanto-occipital extension is necessary to bring the vocal cords within line-of-sight of the mouth. Thus
patients with unstable C1 or C2 injuries might be at more risk from this technique. Direct laryngoscopy
has been shown to disturb the cervical spine both in anaesthetised volunteers (3) and in cadavers (7).
Manual axial in-line stabilisation reduces this movement by 60%. Papers showing no neurological deficit
after direct laryngoscopy have small patient numbers, and there are cases in the literature of quadriplegia
following laryngoscopy without manual stabilization.
Rotando et al evaluated the use of induction agents and muscle relaxants to facilitate intubation and
found them to be safe and effective. If possible, patients requiring tracheal tube intubation should be
anaesthetised unless very cooperative. In the obtunded head injured patient, anaesthesia is vital to
prevent pressor responses to intubation increasing intracranial pressure. Carbon dioxide levels are also
much better controlled in the anaesthetised patient.
The ideal induction agent probably does not exist, and once again it is down to operator experience.
Propofol is not recommended for trauma because of the potential for hypotension (as with most IV
agents) but has many advantages in these patients and is used in many centres. Its ability to provide total
intravenous anaesthesia with good control over the depth of anaesthesia is also very valuable.
Thiopentone (pentothal) requires making up to solution but otherwise is very effective and is the standard
for rapid sequence induction. Etomidate has been reported to produce less cardiovascular depression
than other intravenous induction agents, but this research was done on healthy individuals, and this is not
the case for hypovolaemic patients. In addition, the potential adrenal and immunological suppression
caused by even one bolus of etomidate puts a question mark on its use in these cases. Ketamine is a
very under-used induction agent which maintains cardiovascular stability better than the other
intravenous agents. As a non-competitive NMDA receptor antagonist it has neuroprotective effects. Its
use is currently contraindicated in patients at risk from raised intracranial pressure as it has been shown
to increase cerebral blood flow and so ICP in head injured patients. However, evidence is accumulating
that this may not be the case, especially in hypotensive patients, and its effects on ICP may be
modulated by agents such as propofol.
Awake intubation is also a feasible option and is favoured by some practitioners. It has been shown to be
safe in the patient with cervical spine injury (8). It may be performed via the nasotracheal route, direct oral
laryngoscopy or by fibreoptic technique.
Successful fibreoptic tracheal intubation requires a cooperative patient, a secretion and blood free airway,
a pharynx unrestricted by oedema and adequate supraglottic and infraglottic anaesthesia. Such ideal
conditions often do not exist, and local anaesthetic preparation of the airway is time consuming and might
increase the risk of aspiration.
Failed Intubation
Failed or difficult intubation is always a problem. It is important not to waste time with repeated attempts
at intubation while the patient is desaturating. Alternative methods of securing the airway should be
instituted as soon as a problem is recognised.
Laryngeal Mask Airway (LMA)
The LMA is gaining wider support in the management of patients with cervical spine injury. As well as
maintaining the airway, a tracheal tube (size 6 or less) may be placed, either blindly or via flexible
fibreoptic laryngoscopy. The LMA does not however protect the airway from aspiration, and by acting as
a bolus in the pharynx, may actually relax the lower oesophageal sphincter and increase reflux. It's use
should probably be limited to maintenance of the airway after a failed attempt at intubation.
Combitube
The Combitube is a double lumen tube inserted blindly into the oesophagus or trachea. The position of
the tube is confirmed by the presence of breath sounds or capnography. By inflating one of the two cuffs
present, the lungs may then be ventilated. Problems arise after positioning with definitive securing of a
50
tracheal tube, and again with protection of the airway from aspiration, although stomach suctioning is
possible through the gastric port.
Cricothyroidotomy
The need for a surgical airway should be recognised quickly and performed by an experienced person
without delay. It may be used as a primary airway, with injuries to the pharynx for example, or after failure
of orotracheal intubation. It may be a full surgical approach or via a percutaneous needle
cricothyroidotomy with high flow oxygen. The potential for carbon dioxide retention with this technique
must be remembered and the levels in arterial samples monitored. There are no studies regarding
movement of the neck during cricothyroidotomy, ease of cricothyroidotomy with neck immobilisation, or
neurological deterioration following cricothyroidotomy.
Verification of Tracheal Tube Placement
It is vital that the position of the tube is confirmed to be in the trachea. Clinical methods of verification are
notoriously unreliable, and patients with chest injuries increase the likelihood of mistakes in this area.
Capnography is the gold standard in the operating room to assess tracheal tube position, and this should
probably be transferred to the trauma area too.
Ref:-http://www.trauma.org/archive/anaesthesia/airway.html
220.ANS.D
EXPLANTION:In the normal non-anaesthetised individual, arterial, alveolar and end-tidal pCO2 are often assumed to be
the same, although in reality arterial pCO2 is usually ~ 2-5 mmHg higher than ET-CO2. A significant
difference in end-tidal and arterial pCO2 usually represents the presence of alveolar dead space.
Alveolar dead space (ADSp) represents that part of the inspired gas which passes through the
anatomical dead space but which does not then take part in gas exchange.
Alveolar dead space = physiological dead space – anatomical dead space
ADSp results from ventilated but under-perfused alveoli.
Illustrated by regional differences in alveolar pCO2: basal (V/Q = 0.63, pCO2 42 mmHg) and apical (V/Q
= 3.3, pCO2 28 mmHg).
The presence of ADSp means some alveoli are contributing gas to the expired gas which contain no CO2
(ie. only atmospheric CO2 which is ~ 0.03 %!). Thus the ET-CO2 will be lower than predicted (ie. lower
than the arterial pCO2) because it represents the mixing of gas with various different amounts of CO2.
An increase in ADSp will lead to a decrease in ET-pCO2 (assuming no compensation, such as increased
RR).
Determining alveolar dead space
The Bohr equation can be used to calculate the physiological dead space, by collecting an entire tidal
volume expiration (volume Vt) and determining ‘mixed expired pCO2′ and ‘alveolar pO2′. The Enghoff
modification assumes pACO2 to be the same as paCO2.
Vd = Vt . (PaCO2 - PmeCO2) / PaCO2
The anatomical dead space can be calculated by Fowler’s nitrogen wash-out method. Alveolar dead
space can then be calculated from the two.
Alveolar dead space can also be calculated from the end-tidal CO2:
ADSp = (PaCO2 - PetCO2) / PaCO2
The alveolar gas equation can be used to calculate the alveolar-arterial oxygen difference, which can be
used as an indirect measure of alveolar dead space.
51
A-a gradient = (760 - 47) x FiO2 - pCO2/R + F
Factors increasing dead space
Low cardiac output or hypotension – due to decrsead perfusion of West zone 1. This is the reason for the
observed fall in ET-CO2 in an anaesthetised and hypovolaemic/hypotensive patient.
High inspiratory pressures, particularly if associated with high PEEP – this collapses alveolar capillaries
and again shifts West zone 1.
Pulmonary embolus – due to lack of pulmonary perfusion beyond the emboli.
Posture changes – leading to changes in regional perfusion
In a healthy person breathing room air, the difference between arterial PaCO2 and end-tidal PCO2 is
small.
=> end-tidal PCO2 is about 2~5mmHg lower
The size of this difference is a simple index of the amount of alveolar dead space.
=> as the alveolar dead space volume increases, more relatively CO2-free gas mixes in with gases from
better perfused units, thus lowering the end-tidal PCO2
=> Because PaCO2 is usually very close to PCO2 of the perfused alveoli, increased alveolar dead space
would lower the end-tidal PCO2 and increase the difference between that and arterial PaCO2.
Ref:-http://gasexchange.com/questions/why-is-there-a-difference-betwen-end-tidal-co2-and-arterial-pco2/
221.ANS.D
EXPLANTION:Information specific to: Bupivacaine 50mg/10ml (0.5%) / Adrenaline 50micrograms/10ml (1 in 200,000)
solution for injection ampoules when used in Anaesthesia.
Bupivacaine hydrochloride/Adrenaline acid tartrate is a medicine which is used in local anaesthesia.
The information in this Medicine Guide for Bupivacaine hydrochloride/Adrenaline acid tartrate varies
according to the condition being treated and the particular preparation used.
Bupivacaine hydrochloride/Adrenaline acid tartrate blocks the sensations of pain. It is used to prevent
pain from occurring during or after surgery. It can also be used to prevent pain in certain situations, such
as during labour.
Other information about Bupivacaine hydrochloride/Adrenaline acid tartrate:
a small dose of medicine may be given initially as a test dose
your prescriber will try to find the lowest dose of your medicine which can control your pain
Bupivacaine hydrochloride/Adrenaline acid tartrate is given to you by a healthcare professional. The
person responsible for giving you your medicine will make sure that you get the right dose.
If you feel unwell after having the medicine then talk to your prescriber or someone involved in your
medical care.
Ref:-http://www.nhs.uk/medicine-guides
222.ANS.D
EXPLANTION:A.Incorrect. All non-depolarising neuromuscular-blocking drugs have a quaternary ammonium in their
structure and will not cross the blood–brain barrier.
B.Incorrect. Pancuronium is a long-acting blocking drug and is used for procedures taking longer than
90 min.
C.Incorrect. Short-acting non-depolarising blocking drugs such as rocuronium or mivacurium are viable
alternatives.
D.Incorrect. More than 90% of receptors need to be occupied to produce a complete block of skeletal
muscle contractility.
E.Correct. Atracurium is one of the most potent neuromuscular-blocking drugs causing the release of
histamine.
52
Compendium: drugs acting at the neuromuscular junction
Drug Kinetics (half-life)
Comments
Acetylcholinesterase inhibitors
Distigmine, edrophonium, pyridostigmine
See Ch. 28
Neostigmine
See Ch. 28
Given intravenously to reverse the effects of a non-depolarising blocker
(with atropine to minimise effects of enhanced acetylcholine on the parasympathetic system); also used
in myasthenia gravis (Ch. 28)
Non-depolarising blockers
All are used for muscle relaxation for surgery; all have negligible oral absorption and are given by
intravenous injection or infusion.
Atracurium
Spontaneous hydrolysis; very rapidly breaks down within blood (0.3 h); unaffected by
hepatic or renal failure Also used for muscle relaxation during intensive care; a complex mixture of 10
isomers; cardiovascular effects due to histamine release
Cisatracurium Spontaneous degradation (0.5 h); unaffected by hepatic or renal failure Also used for
muscle relaxation during intensive care; a single isomer of atracurium; does not cause histamine release
Mivacurium
Hydrolysed by pseudocholinesterase (2–60 min); prolonged effect in rare individuals with
genetic deficiency of this enzyme
Consists of three isomers, two of which are responsible for the
clinical response
Pancuronium Hydrolysed to an active product (0.5 h); eliminated by renal and biliary routes
Often
used for long-term muscle relaxation during mechanical ventilation in intensive care; sympathomimetic
effects can cause tachycardia and hypertension
Rocuronium
Hepatic metabolism and renal elimination (1.2 h) Also used for muscle relaxation during
intensive care; most rapid onset of action of drugs in this class; minimal cardiovascular effects
Vecuronium
Hepatic metabolism; biliary excretion (1 h)
Lacks cardiovascular effects
Depolarising blocker
Suxamethonium (succinylcholine)
Hydrolysed by pseudocholinesterase (2–5 min); prolonged effect
in the rare individuals with genetic deficiency of this enzyme
Given intravenously for blockade of
rapid onset and short duration of action (e.g. intubation); paralysis preceded by painful fasciculations.
Ref:-Medical Pharmacology and Therapeutics
by Derek G. Waller, Anthony P. Sampson, Andrew G. Renwick, and Keith Hillier
4th edition published by Elsevier
223.ANS.A
EXPLANTION:1.True. Changing the circulating air in the operating theatre 15–20 times per hour is one of the most
effective methods of reducing pollution. An unventilated theatre is about four times more polluted
compared to a properly ventilated one.
2.True. The patient should be protected against excessive positive and negative pressures being applied
by the scavenging system. Excessive positive pressure puts the patient under the risk of barotrauma.
Excessive negative pressure causes the reservoir in the breathing system to collapse thus leading to
incorrect performance of the breathing system.
3.True. In the UK, the maximum accepted concentration of nitrous oxide is 100 ppm over an 8-hour timeweighted average.
4.False. Because of the nature of the flow of the exhaled gases, the scavenging system should be
capable of tolerating high and variable gas flows. The flow of exhaled gases is very variable during both
spontaneous and controlled ventilation. An ordinary vacuum pump might not be capable of coping with
such variable flows, from 30 to 120 L/min. The active scavenging system is a high-flow, low-pressure
system. A pressure of −0.5 cm H2O to the patient breathing system is needed. This cannot be achieved
with an ordinary vacuum pump (low-flow, high-pressure system).
Ref:-Essentials of Anaesthetic Equipment
by Baha Al-Shaikh and Simon Stacey
4th edition published by Elsevier
224.ANS.B
53
EXPLANTION:Alveolar partial pressure of a volatile anesthetic, which ultimately determines the depth of general
anesthesia, is determined by the relative rates of input to removal of the anesthetic gases to and from the
alveoli. Removal of anesthetic gases from the alveoli is accomplished by uptake into the pulmonary
venous blood, which is most dependent upon an alveolar partial pressure difference. During the initial
moments of inhalation of an anesthetic gas, there is no volatile anesthetic in the alveoli to create this
partial pressure gradient. Therefore, uptake for all volatile anesthetic gases will be minimal until the
resultant rapid increase in alveolar partial pressure establishes a sufficient alveolar-to-venous partial
pressure gradient to promote uptake of the anesthetic gas into the pulmonary venous blood. This will
occur in spite of other factors, which are discussed in the explanation to question 333
Ref:-(Miller: Anesthesia, ed 6, pp 133-135).
225.ANS.D
EXPLANTION:At concentrations of 1 MAC or less, volatile anesthetics, as well as the inhaled anesthetic N2O, will
produce dose-dependent increases in the respiratory rate in spontaneously breathing patients. This trend
continues at concentrations greater than 1 MAC for all of the inhaled anesthetics except isoflurane. With
the exception of N2O, the evidence suggests this effect is caused by direct activation of the respiratory
center in the central nervous system rather than by stimulating pulmonary stretch receptors. Additionally,
volatile anesthetics decrease Vt and significantly alter the breathing pattern from the normal awake
pattern of intermittent deep breaths separated by varying time intervals to one of rapid, shallow, regular,
and rhythmic breathing
Ref:-(Miller: Anesthesia, ed 6, pp 170-171; Stoelting Pharmacology and Physiology in Anesthetic
Practice, ed 4, p 60).
226.ANS.A
EXPLANTION:Urticaria pigmentosa (UP) is a skin condition that causes lesions and itching. It is characterized by the
presence of too many mast cells in the skin.This disease is most commonly seen in infants and children
but adults can be affected as well. The main symptom is dark-colored lesions on the skin. The lesions
may be very itchy and difficult not to scratch. When rubbed or scratched, they respond with a Darier’s
sign. A Darier’s sign looks similar to hives. This is caused by the release of histamine from the mast
cells.In most children, UP goes away by puberty. Complications are typically only seen in older children or
adults. Rarely, UP may evolve into systemic mastocytosis in an adult. In this condition, mast cells can
build up in other organs of the body. In extremely rare cases, mast cell leukemia or mast cell sarcoma
may result.
Ref:-Dermatology: Illustrated Study Guide By Sima Jain
227.ANS.C
EXPLANTION:First described by Brocq in 1907, using the term eczema craquelé, asteatotic dermatitis is characterized
by pruritic, dry, cracked, and polygonally fissured skin with irregular scaling. It most commonly occurs on
the shins of elderly patients, but it may occur on the hands and the trunk.
superficial bleeding and fissures can occur as the epidermis loses water, as it splits, and as it cracks
deeply enough to disrupt papillary dermal capillaries. The inflammation can be associated with
asymmetric leg edema. Eczema with increased lichenification occasionally supervenes as patients rub
and scratch the pruritic areas.
The eruption can be generalized or localized. Generalized asteatosis is a distinct entity and should
provoke a search for possible associated diseases. Guillet divides the localized forms into 4 types:
Asteatotic eczema of the lower extremities in elderly persons secondary to aging, dehydrated skin, and
malnutrition
Cracked erythema secondary to irritant contact dermatitis from soaps or detergents
54
Eczema craquelé in areas in which corticosteroid therapy was discontinued
Asteatotic eczema in neurologic disorders
Initially, excess water loss from the epidermis results in dehydration of the stratum corneum with upward
curling of corneocytes. The outer keratin layers require 10-20% water concentration to maintain their
integrity. A significant decrease in free fatty acids in the stratum corneum is present in people with
asteatotic dermatitis. Stratum corneum lipids act as water modulators, and cutaneous loss of these lipids
can increase transepidermal water loss to 75 times that of healthy skin
During the winter months, an elderly person classically presents with pruritic and dry skin with dermatitis
on the pretibial areas. Sometimes, the dysesthesia may be described as a pinprick or biting sensation.
Asking the patient about pertinent controllable factors, such as the following, is important:
Frequency of bathing, showering, and cleansing, and which soaps and cleansers are in contact with the
skin
Types of skin lubricants used, and method and frequency of application
Diet
Medications
Types of clothing worn (Wool may cause irritation.)
The source, the type, and the temperature of heat that may alter the humidity of the environment
If the eruption persists despite therapy, behavioral changes, and treatment compliance, allergic contact
dermatitis and irritant contact dermatitis and internal malignancy may require investigation.
Ref:-Dermatology: Illustrated Study Guide By Sima Jain
228.ANS.A
EXPLANTION:Nails
Onychodystrophy is any abnormality of nails.
Nails may show pitting, including thimble pitting, as in psoriasis when they have numerous tiny
indentations like a thimble.
Onychomadesis is loosening and shedding of nails.
Onychogryphosis is an abnormal condition of the nails characterised by marked hypertrophy and
increased curvature.
Onychoschizia is a condition of the nails marked by lamination in two or more layers and by scaling away
in thin flakes.
Koilonychia is spoon nails and is a feature of iron deficiency.
Clubbing involves increased curvature in both directions. It may be congenital or indicate other diseases.
Subungual means under the nails.
Pterygium is a forward growth of the cuticle over the nail. It is also a triangular fleshy mass of thickened
conjunctiva at the inner side of the eyeball, covering part of the cornea, and causing a disturbance of
vision.
Ref:-Dermatology: Illustrated Study Guide By Sima Jain
229.ANS.A
EXPLANTION:i. The most likely diagnosis is seborrhoeic dermatitis.
Ii. The differential diagnosis should include psoriasis, allergic contact dermatitis to a topical application,
and atopic dermatitis, although this latter would be unusual occurring for the first time in an adult.
Psoriasis is not usually itchy, but certainly can be. Other disorders such as rosacea and lupus
erythematosus are seldom itchy.
Iii. If there is doubt concerning the clinical diagnosis, it is worth performing a biopsy as although an
indeterminate result may not be helpful, a positive result would be conclusive. If a 3 or 4 mm punch
biopsy is performed without sutures being inserted, scarring will not be a problem. Patch testing to
cosmetics or medicaments will be appropriate if allergic contact dermatitis is suspected. If the patient has
severeseborrhoeic dermatitis occurring for the first time, the possibility of human immunodeficiency virus
(HIV) disease or other form of immunosuppression should be considered and the appropriate blood tests
performed.
55
Ref:- Dermatology: Illustrated Study Guide By Sima Jain
230.ANS.D
EXPLANTION:The lesion is a plantar wart. Callosities are found at sites of friction, and are not as well marginated.
Corns are mostly found on the tarsal arch and do not have tiny black dots on the surface as do plantar
warts.
Epidemiology
More common in adolescents and young adults
Plantar warts responsible for a third of warts
Pathophysiology
Human papillomavirus types 1,2 and 4 most common
Predisposing factors
Excessive sweating
Repetitive microtrauma to foot (e.g. sports)
Public showers
Types
Classic Plantar Wart
Mosaic Wart (coalescence of multiple lesions)
Myrmecia (refers to anthill-like lesion)
Epidermal lesion extends deeply under skin surface
Usually associated with HPV Type 1
Symptoms
Pain at heel and metatarsal heads (pressure points)
Worse with walking (feels like pebble in shoe)
Signs
Location
Pressure points on metatarsal heads or heel
Characteristics
Skin-colored hyperkeratotic Papules or Plaques
Minute, hemorrhagic puncta (visible after paring)
Represent thrombosed capillaries
Accentuated by
Soak in warm water 15-20 minutes to make more visible
Differential diagnosis
Corn (or callus)
Yellow-white firm glistening central kernel (pared)
Painless on lateral pressure
Retain normal fingerprint lines (warts do not)
Contrast with pinpoint blood vessels of plantar warts
Stress Fracture
Foreign body reaction
Lichen planus
Black Dot Heel (sheering trauma in sports)
Epithioma cuniculatum (verrucous carcinoma)
Ref:-Dermatology: Illustrated Study Guide By Sima Jain
231.ANS.D
EXPLANTION:This patient is showing evidence of brief psychotic disorder. This disorder is characterized by psychotic
symptoms lasting >1 day, but <1 month; she has had symptoms for the past 2 weeks. Also, the stress of
failing the exam is likely to be a precipitating psychosocial factor in this patient. Schizoaffective disorder is
characterized by symptoms of both a mood disorder and schizophrenia, as well as psychotic symptoms
that occur even in the absence of mood symptoms, and lifelong social and occupational impairment. In
schizophrenia, psychotic and residual symptoms last 6 months, and there is lifelong social and
56
occupational impairment. Schizophreniform disorder is characterized by psychotic and residual symptoms
lasting 1–6 months. In delusional disorder, which often lasts for years, there is a fixed, non-bizarre
delusional system; few, if any, other thought disorders; and relatively normal social and occupational
functioning. In shared psychotic disorder, a person develops the same delusion as a person with
delusional disorder with whom they are in a close relationship. Psychosis due to a general medical
condition involves psychotic symptoms occurring as a result of physical illness (see the TBQ).
Ref:-BRS Behavioral Science
by Barbara Fadem
6th edition published by Wolters Kluwer
232.ANS.C
EXPLANTION:Tardive dyskinesia (TD) is a disorder characterized by involuntary choreoathetoid movements of the face,
trunk, and extremities. Tardive dyskinesia is associated with prolonged use of medications that block
dopamine receptors, most commonly antipsychotic medications. Typical antipsychotic medications (such
as perphenazine) and, in particular, high-potency drugs carry the highest risk of TD. Atypical
antipsychotics are thought to be less likely to cause this disorder and clozapine is used in the treatment of
TD.
Ref:-(Hales, 3/e, pp 1069–1071.)
233.ANS.C
EXPLANTION:An hallucination is the perception of a stimulus when, in fact, no sensory stimulus is present.
Hallucinations can be auditory, visual, tactile, gustatory, olfactory, or kinesthetic (body movements).
Auditory hallucinations are most commonly associated with psychotic illness, whereas visual, tactile,
gustatory, and olfactory hallucinations often are associated with neurologic disorders. A delusion is an
erroneous unshakable belief, and an idea of reference is a form of delusion.
Ref:-Psychiatry,J. Clive Spiegel, MD SECOND EDITION,(Sadock, 7/e, p 810.)
234.ANS.B
EXPLANTION:The capacity to generalize
and to formulate concepts is called abstract thinking. The inability to abstract
is called concreteness and is seen in organic disorders and sometimes in schizophrenia. Abstract
thinking is commonly assessed by testing similarities,
differences, and the meaning of proverbs. Intellectualization and rationalization
are unconscious defenses, while delusional thinking refers to
fixed beliefs with no basis in reality.
Ref:-Psychiatry,J. Clive Spiegel, MD SECOND EDITION,(Hales, 3/e, pp 219–220.)
235.ANS.A
EXPLANTION:The patient’s persecutory delusions and disorganized thinking could suggest a psychotic disorder
such as schizophrenia or brief reactive psychosis, but fluctuations in consciousness and disorientation
are typically found in delirium. Memory, language, and sleep-wake cycle disturbances are also typical of
delirium. Delusions, hallucinations, illusions, and misperceptions are also common. The causes of
delirium are many and include metabolic encephalopathies, such as the hyperglycemic encephalopathy
experienced by the patient in the vignette; intoxications with drugs and poisons; withdrawal syndromes;
head trauma; epilepsy; neoplasms; vascular disorders; allergic reactions and
injuries caused by physical agents (heat, cold, radiation).
Ref:-Psychiatry,J. Clive Spiegel, MD SECOND EDITION,(Yudofsky, 3/e, pp 455–459.)
236.ANS.A
57
EXPLANTION:Intermittent explosive
disorder is characterized by discrete episodes of aggression and destructiveness that are out of
proportion with the precipitant stressor. EEG may help to clarify the diagnosis, since 55% of patients have
EEG abnormalities as well as learning disabilities, a history of attention deficit hyperactivity disorder
(ADHD), and abnormal neuropsychological test results. EEG may also be helpful in diagnosing the rare
cases of violence that occur during or after a complex partial seizure and specific types of delirium and
dementing disorders (for example, Creutzfeldt-Jakob dementia and delirium induced by hepatic
encephalopathy are both associated with very specific EEG patterns). Frotteurism (a paraphilia), panic
disorder, social phobia, and bipolar disorder are not associated with EEG changes.
Ref:-Psychiatry,J. Clive Spiegel, MD SECOND EDITION,(Hales, 3/e, pp 292–293, 776, 1416.)
237.ANS.D
EXPLANTION:Lumbar puncture is not part of the routine work-up for classic Alzheimer’s dementia. It should be
considered when infection and carcinomatous meningitis are considerations.A lumbar puncture is not a
routine test in the diagnosis of Alzheimer's. It can be performed to exclude people showing apparent
dementia type symptoms who are suffering from rare diseases of the central nervous system. These
diseases can be caused by infections such as TB (tuberculosis), syphilis and certain types of fungal
inflammations.
Ref:-http://alzheimers.about.com/od/diagnosisissues/a/lumbar_puncture.htm
238.ANS.C
EXPLANTION:Generalized anxiety disorder is characterized by the presence of excessive worry and associated
physical symptoms (in this patient, sleep problems, concentration difficulties, and fatigue) that persist
most days for at least 6 months.
Ref:-www.uptodate.com
239.ANS.B
EXPLANTION:Contrasts with phobic disorders where little anxiety may be experienced in absence of phobic stimulus.
Ref:-Textbook of Psychiatry
by Basant K. Puri and Ian H. Treasaden
3rd edition published by Elsevier
240.ANS.E
EXPLANTION:Parapraxis is a term used to describe an error of memory, speech, writing, reading or action that may be
due to the interference of repressed thoughts and unconscious features of the individual’s personality. It
is referred to in common culture as a ‘slip of the tongue’ or a ‘Freudian slip’.
Ref:-Psychiatry
by Katie Marwick and Steven Birrell
4th edition published by Elsevier
241.A
242.C
243.D
244.D
246.ANS.C
EXPLANTION:-
58
245.D
Chest radiograph of a 58-year-old man with malignant melanoma (note surgical clips in right lower neck)
shows multiple pulmonary nodules of varying sizes consistent with metastatic disease. There is also a
small right basal effusion.
Pulmonary metastasis is seen in 20-54% of extrathoracic malignancies.[1] Lungs are the second most
frequent site of metastases from extrathoracic malignancies. Twenty percent of metastatic disease is
isolated to the lungs.[1] The development of pulmonary metastases in patients with known malignancies
indicates disseminated disease and places the patient in stage IV in TNM (tumor, node metastasis)
staging systems. This typically implies an adverse prognosis and alters the management plan. Imaging
plays an important role in the screening and detection of pulmonary metastases. Imaging guidance is
also used in histological confirmation of metastatic disease. In patients with poor cardiorespiratory
function and comorbidities, imaging-guided thermal ablation procedures are an effective alternative to
surgical resection to improve the survival.
Chest radiography (CXR) is the initial imaging modality used in the detection of suspected pulmonary
metastasis in patients with known malignancies. Chest CT scanning without contrast is more sensitive
than CXR. For patients with bone or soft-tissue sarcoma, malignant melanoma, and head and neck
carcinoma, CT scanning of the chest should be performed as an initial evaluation. In patients with primary
renal or testicular cancer, chest CT scanning performed should be performed based on the presence of
metastatic disease elsewhere. CT guidance is often required for obtaining samples from a suspected
metastatic disease. Several thermal ablation options are available for treatment of pulmonary
metastases, which is performed under CT guidance.
Whereas the differential diagnosis of pulmonary masses includes some benign lesions, the majority of
lesions this size (5 cm) are primary lung cancers. The size of the lesion, as well as the pattern of
calcification, can be useful in estimating the likelihood of malignancy. Seventy-five percent of all lung
lesions 3 cm or greater are malignant
Ref:-http://emedicine.medscape.com/article/358090-overview
247.ANS.D
EXPLANTION:Digital chest x-ray of female patient shows multiple calcified pleural plaques caused by asbestos
exposure. Plaques are also seen along the diaphragm.
Effusions in benign pleural disease are usually seen within 10 years of exposure, and they are often
asymptomatic, small (less than 500 mL), and exudative.
In parenchymal disease, asbestos-induced pulmonary fibrosis usually occurs 20 to 30 years after high
levels of exposure. It progresses even after exposure cessation and causes progressive restrictive lung
function. Asbestosis appears as interstitial reticulation and "shaggy heart" on chest x-rays, while
subpleural reticulation, branching linear peripheral opacities, and "honeycombing" can be seen on highresolution CT, they pointed out. Parenchymal bands and subpleural curvilinear opacities are less specific
CT features. This condition is compensable.
Malignant mesothelioma also tends to emerge 20 to 40 years postexposure. Extensive smooth or
irregular pleural thickening occurs and circumferential involvement distinguishes the condition from
benign thickening. It can present as massive effusion with minimal pleural thickening and is locally
invasive. There is a need to differentiate it from pseudomesothelioma due to metastatic tumor. Again, it is
compensable, according to the Cardiff group
As the patient probably has had significant exposure (he is a former shipyard worker), the most likely
cause for the lower lobe interstitial infiltrates as well as pleural plaques is asbestos. Cardiogenic
pulmonary edema does not cause pleural calcifications, nor do any of the other diagnoses listed.
Ref:-http://www.auntminnieeurope.com/index.aspx?sec=ser&sub=def&pag=dis&ItemID=607262
248.ANS.D
EXPLANTION:Findings:
A pre-existing cavity containing a mycetoma is present.
59
Rationale:
A: Incorrect.
B: Incorrect.
C: Incorrect.
D: Aspergilloma or mycetoma or a fungus ball consists of aspergillus hyphae, mucus and cellular debris
which is present in a pre-existing cavity. It grows as a saprophytic organism and as a rule is noninvasive.
Thus, no treatment is necessary in asymptomatic individuals. However, in some individuals severe
hemophysis may occur and be life threatening. In those individuals, conventional angiography is
performed for embolization of bronchial arteries which supply the cavity.
Ref:-Fundamentals of Diagnostic Radiology,by William E. Brant ,Mar 2016
249.ANS.A
EXPLANTION:Findings:
Chest radiograph shows a small left hyperlucent lung. CT scan of the chest shows decreased attenuation
and vascularity in the left lung. In addition, mild bronchiectasis is also noted in the left lung.
Rationale:
A: The radiographic findings present in this case are typical of Swyer-James syndrome. The syndrome is
believed to be related to a viral bronchiolitis in childhood. This manifests as a small hyperlucent lobe or
lung. The hyperlucency is due to bronchiolar obliteration. Air-trapping on expiration scan is also noted.
B: Incorrect.
C: Incorrect.
D: Incorrect.
Ref:-Fundamentals of Diagnostic Radiology,by William E. Brant ,Mar 2017
250.ANS.B
EXPLANTION:To see an air-fluid level, the x-ray beam must be directed horizontally ─ parallel to the floor. Air-fluid
levels will not be visible on a supine radiograph such as this (oval).
On an upright or decubitus view of the abdomen, there is almost always an air-fluid level in the stomach,
there may normally be two or three air- fluid levels in the small bowel and there are usually no or very few
air- fluid levels in the colon.
The supine view is used to assess the overall appearance of the bowel gas pattern, to identify the
presence or absence of calcifications and to identify the presence of soft tissue masses.
Ref:-Learning Radiology: Recognizing the Basics by William Herring 2nd edition published by Elsevier
251.ANS.C
EXPLANTION:The star is in the normal stomach.
Just inferior to the stomach is the transverse colon (T). Haustral markings in the colon usually do not
extend completely across the large bowel from one wall to the other.
If they do connect one wall with another, haustral markings are spaced more widely apart than the
valvulae conniventes of the small bowel.
60
Large bowel characteristically is placed peripherally around the outer aspects of the abdomen (arrows)
forming a picture frame for the small bowel in the center.
Ref:-Learning Radiology: Recognizing the Basics by William Herring 2nd edition published by Elsevier
252.ANS.A
EXPLANTION:MRI demonstrates typical findings of a large left sided acoustic schwannoma with cystic change - a
common finding in larger schwannomas. Sections show a variably cellular tumour with compact areas of
fascicular growth pattern and prominent nuclear palisading
Brain, left acoustic lesion: Schwannoma.
Ref:-http://radiopaedia.org/cases/acoustic-schwannoma-large-with-cystic-change
253.ANS.A
EXPLANTION:POSTERIOR CEREBRAL ARTERIES — Most posterior cerebral artery (PCA) territory infarcts are due to
embolism from the heart, aorta, or vertebral arteries. Atherosclerosis and dissection of the PCAs is not
common. Visual loss — The most frequent finding in patients with PCA territory infarction is a
hemianopia. At times, the central or medial part of the field is spared, so-called macular sparing. A
superior quadrant field defect results if the infarct involves just the lower bank of the calcarine fissure (the
lingual gyrus). An inferior quadrantanopia results if the lesion affects the cuneus on the upper bank of the
calcarine fissure. When the full PCA territory is involved, visual neglect can accompany the hemianopia.
However, patients are aware of the visual defect when infarcts are restricted to the striate cortex and do
not extend into the adjacent parietal cortex. The visual defect is often described as a void, blackness, or a
limitation of vision to one side, and patients usually recognize that they must focus extra attention to the
hemianopic field. When given written material or pictures, patients with hemianopia due to occipital lobe
infarction are able to see and interpret stimuli normally, although it may take somewhat longer to explore
the hemianopic visual field.
Ref:-http://radiopaedia.org/
254.ANS.D
EXPLANTION:Osteosarcoma is a highly malignant bone tumor characterized by formation of bone tissue by tumor cells.
It is most frequent in adolescents between the ages of 10 and 20 years. Almost two thirds of cases of
osteosarcoma exhibit mutations in the retinoblastoma (Rb) gene, and many tumors also contain
mutations in p53. Often, the periosteum produces an incomplete rim of reactive bone adjacent to the site
where it is lifted from the cortical surface by the tumor. When this appears on an X-ray as a shell of bone
intersecting the cortex at one end and open at the other end, it is referred to as Codman triangle. A
“sunburst” periosteal reaction is often superimposed. Mutations in the fibroblast growth factor receptor
gene (choice C) are a cause of achondroplasia. Deregulation of cyclins (choices A and B) and cyclindependent kinases are associated with several neoplasms.
61
Diagnosis: Osteosarcoma
Ref:-Rubin’s Pathology: Clinicopathologic Foundations of Medicine
by Raphael Rubin and David S. Strayer
Rubin’s Pathology: Clinicopathologic Foundations of Medicine by Raphael Rubin and David S. Strayer
6th edition published by Wolters Kluwer
255.ANS.B
EXPLANTION:Kyphosis is characterized by a “hunchback” due to an abnormal increase in curvature of the thoracic
region of the vertebral column. Lordosis, or “swayback,” is an increase in lumbar curvature of the spine.
Lordosis can be physiologic, such as seen in a pregnant woman. Scoliosis is a lateral curvature of the
spine with rotation of the vertebrae. Spina bifida is a neural tube defect characterized by failure of closure
of the vertebral arch. Osteoarthritis is a degenerative disorder that affects the articular cartilage of joints
and is not specifically related to the thoracic region of the spine.
Ref:-GAS 77; GA 2, 26
256.D 257.B 258.C 259.A 260.B 261.C 262.B 263.B 264.D 265.A 266.C 267.D 268.A
269.D 270.D 271.A 272.C
273 Ans : D Heller’s opration is the best. Botulinum toxin is the least effective.
274.C 275.A 276.C 277.C 278.A 279.B 280.D 281.C 282.D 283.A 284.C 285.D 286.D
287.A 288.A
289.Ans. C: Paradoxic aciduria
 Usual victim in surgical ward is pt suffering from pyloric stenosis.
 Hypochloremic alkalosis occurs
 Hypokalemia and alkalosis usually co-exist.
 To compensate hypokalemia, renal tubules absorb K in exchange of H ions (Paradoxic aciduria).
Hypokalemic, hypochloremic alkalosis is corrected by giving NaCl and KCl.
290. D:
Tumour does not spread to LN. PO radiation to chest wall is good.
Small, benign, phyllodes tumour of young females is treated by lumpectomy –
enucleation is not enough (1cm margin).
291.D 292.D 293.C 294.D 295.A 296.C 297.C 298.D 299.C 300.A
62